Download as pdf or txt
Download as pdf or txt
You are on page 1of 106

It is against copyright law to take photos or distribute this content May 2023

1 A 52 year old man attends his GP with a rash across his stomach and lower
back that started several days ago He denies any itching, soreness or
burning He is otherwise well in himself On examination, scaly red oval
patches are seen across the abdomen and all over his back A larger
circular patch is also noticed on his upper back Below are pictures of the
lesions

By .ames HMIBR.MC . CC SY-SA 3 0

What is the MOST likely diagnosis?


A Pityriasis versicolor
B Pityriasis rosea
C Atopic dermatitis
D Pityriasis alba
E. Discoid eczema
it is against copyright few to take photos or distribute this content May 2023

2. A 73 year old man is admitted to the cardiac care unit following a diagnosis
of an ST-elevation myocardial infarction. During the coronary angiogram, he
develops ventricular fibrillation What is the SINGLE most appropriate
management ?
A IV amiodarone
B Synchronised DC shock
C . Unsynchronised DC shock
D IV magnesium sulphate
E IV adrenaline

3. A 76 year old woman with reduced ejection fraction heart failure and atrial
fibrillation attends the Cardiology clinic for a review as she has developed
worsening shortness of breath and ankle swelling over the past few months
She was diagnosed with heart failure 8 years ago. She is currently taking
ramipril , bisoproiol, nvaroxaban and atofvastatin What medication should
be added? 5
A Warfarin
B Amfodipine
C. Losartan
D Digoxin
E Amiodarone

it is against copynght taw to lake photos or distnbute content May 2023


4
tt is against copyright taw to take photos or distribute this content May 2023

4. A 16 year old girl presents to the Accident and Emergency Department with
persistent nausea and vomiting. The senior house officer (SHO) on- call
offers an antiemetic and administers metoclopramide 10mg intravenously.
The girl suddenly develops a stiff neck and double vision The A&E registrar
quickly recognises this as a complication of metoclopramide and
administers procyclidme What is the mechanism of action of
metoclopramrde?
A Dopamine agonist
B Dopamine antagonist
C. Muscarinic antagonist
D . Serotonin 5 -HT 3 receptor antagonist
E Serotonin 5-HT 3 receptor agonist
vcP
5 A drunken 21 year old diabetic man rsJbhbught in by the police to the
Emergency Department after a fight in a nightclub, complaining of being
bitten On examination, a bftetnark is noted on his left hand, with an
intercanine distance of more than 3 cm. The bite has broken the skin but is
not bleeding There is no associated erythema, swelling or discharge. His
observations are normal. He is up to date with his vaccination schedule.
What is the SINGLE most appropriate initial management?
A Wound microscopy, culture and sensitivity (MC&S)
B Co -amoxiclav
C Tetanus vaccination
D Wound irrigation only
E Co -amoxiciav + tetanus vaccination

ft is against copyright (aw to photos csr distnbute tftfs co/rfenf . May 2023
5
it is against copyright (aw to lake photos or distribute this content May 2023

6. A 46 year old man presents to his General Practitioner with weight gain,
particularly around his neck and stomach. He also finds it increasingly
difficult to get himself out of a chair. His background includes a basal cell
skin carcinoma removal 3 years ago He also smokes 15 cigarettes a day. A
blood pressure reading in clinic is 159/105 mmHg. A late night salivary
cortisol and 24 -hour urinary cortisol are found to be high Serum ACTH is
0.7 pmol/L [2 2 - 13 3] What is the SINGLE most likely underlying caused
A Pituitary adenoma
B Paraneoplastic syndrome
C Exogenous steroid
D Primary hypothalamic disease
E Adrenal adenoma

AS
7 A 26 year old woman comes in to see her GP after being fired from multiple
jobs Although she was initially excited and motivated upon joining her most
recent company, she began making silly mistakes and felt bored and
unmotivated after a few months of employment. She does not follow
through with instructions and has difficulty organising tasks. She has
misplaced her work laptop on several occasions At home, she reports
things are generally “ alright", although she does struggle to keep the house
clean consistently and to remember her appointments. She fidgets with her
hands often She has occasions where she interrupts people's conversation
because she cannot wait her turn What is the SINGLE most likely
diagnosis?
A Mania
B Anxiety
C Attention deficit hyperactivity disorder (ADHD)
D Chronic fatigue syndrome
E . Autism spectrum

it rs agawisf copyright (aw to take photos or distribute this content May 2023
6
tt is against copyright law to take photos or distribute this content. May 2023

8. An 19 year old man is found in his university accommodation with a suicide


note it states he wishes to end his life and that he has taken 36
,

paracetamol tablets with a mixture of alcohol and sleeping tablets. He was


last seen by his room mate six hours ago at dinner An ambulance is called
and he is seen in A&E within an hour On assessment, he is disoriented and
vomiting. What is the SINGLE next most appropriate action?
A . Start N-acetylcysteme immediately
B Activated charcoal
C Gastric lavage
D Check paracetamol level 4 hours after arrivai
E. Check paracetamol level and start N-acetylcysteine if above treatment
line

9. A 60 year old man presents with a 3 month history of epigastric pain and
bloating that worsens upon lying fiat awf after eating certain foods He has
presented previously due to similar symptoms and was initially started on a
proton pump inhibitor On further questioning, he denies any difficulty
swallowing or altered bowel habits. He has noticed that in this time , he has
unintentionally lost around 6 kg in weight. What is the SINGLE most
appropriate management?
A Helicobacter pylori breath test
B Urgent oesophagogastroduodenoscopy
C . Non-urgent oesophagogastroduodenoscopy
D Dietary and lifestyle advice
E Barium swallow

tt is against copyright iaw to Sake photos or cfr&ribute tois content May 2023
7
It is against copyright law to lake photos or distribute this content May 2023

10 A 30 year old woman attends tier General Practice for advice regarding
tier holiday next week. She is currently 34 weeks pregnant and has read
about the risk of blood clots after airplane travel. Her husband booked the
holiday to France as a surprise, which is 2 hours long. She has a normal
body mass index with no other risk factors for venous thrombosis What is
the SINGLE most appropriate advice?
A Start aspirin on day of flight
B Start low molecular weight heparin on day of flight
C Advise against flying at this gestation
D Provide advice regarding hydration, in-seat exercises and
compression stockings
E Request an obstetric review for advice

11 . A 39 year old man has been feeling palpitations and shortness of breath
over the past few days He has episodes where he feels his heart beats
irregularly and faster than normal. His blood pressure is 150/90 mmHg and
pulse rate is 69 beats/ minute in the clinic. He is afebrile. He drinks 21 units
of alcohol a week. He uses ibuprofen occasionally for a previous shoulder
injury What is the SINGLE most likely cause of his shortness of breath and
palpitations?
A Bacterial infection
B Viral infection
C Hypertension
D Alcohol
E Ibuprofen

It is copyright taw to lake photos or distribute (his content May 2023


3
it is against copyright taw to lake photos or distribute hits content . May 2023

12. A 32 year old woman presents with intermittent diarrhoea and bloating.
She denies any rectal bleeding or weight loss Her past medica] history
includes vitiligo. On examination , there are pale mucous membranes but no
abnormal clinical signs within the abdomen. Investigations reveal:

Haemoglobin 112 g/L ( 115-160)


Platelets 330 x 10*/L (150- 400)

Anti -tissue transglutaminase IgA Negative

Total IgA Undetectable

What is the SINGLE most appropriate next step?


A . Duodenal biopsy
B Anti-gliadm IgG
C Anti-tissue transglutaminase IgG
D Bore marrow biopsy
E Oral ferrous sulphate

13 A 4 year old girl has an atypical urinary tract infection with septicaemia
She undergoes an abdominal ultrasound scan during her hospital stay
which is reported as normal She is Ireated with intravenous antibiotics and
recovers after 4 days What is the SINGLE most appropnate action?
A Perform a dimercaptosuccinic acid (DMSA) scan in 4 weeks after the
infection
B Perform a dimercaptosuccinic acid {DMSA) scan in 4 months after the
infection
C . Repeat the ultrasound scan in 6 weeks
D Perform a micturating cystogram (MCUG)
E . No further fnvestigations required

it is against copyright law to take photos or distribute this content . May 2023
9
it is against copyright law to lake photos or distribute this content . May 2023

14 A 45 year old Afro-caribbean man returns to his General Practitioner


after doing a week of ambulatory blood pressure readings. Me has a
background of type 2 diabetes mellitus. asthma and class II obesity What
class of antihypertensives should be offered first-line ?
A ACE -inhibitor
B Calcium channel blocker
C Beta blocker
D Angiotensin receptor blocker
E Bendraft umethiazide

15 A 31 year oEd woman has had palpitations over the past 2 weeks. She
denies any chest pain or shortness of breath. Her heart rate is 128
beats/minute An ECG is performed and shows that she has sinus
tachycardia. She has no significant medical history What is the SINGLE
most appropriate action?
A 24 -hour tape
B . Thyroid function test
C. Chest x- ray
D Echocardiogram
E. Troponin levels

It is against copyright law to take photos or distribute ffris content May 2023
10
tt is against copyright taw to tofce photos or distribute this content May 2023

16 A 22 year old man presents to A&E with sudden onset testicular pain and
swelling. He also has associated lower abdominal aching pain and has
vomited twice. On examination, the left testis lies horizontally high within
the neck of the scrotum. The right testis is normal. Pain is not relieved by
elevating the affected testis What is the SINGLE most likely diagnosis ?
A Testicular torsion
B Acute epididymitis
C Strangulated inguinal hernia
D. Varicocele
E Epididymo-orchitis*

&
$

tt is against copyright taw to take photos or distribute this content . May 2023
11
it is against copyright lew to lake photos or distribute this content. May 2023

17 A 63 year old woman has sudden pain in her left calf and foot. There is
swelling in her calf. She has been bed bound for the past 2 weeks
recovering from a total hysterectomy and bilateral salpingo-oophorectomy
She denies any shortness of breath. There is pitting oedema confined to
her left leg. There is noticeable swelling of the calf and foot She has a
temperature of 36 7 X and a heart rate of 70 beats/minute A picture of the
leg is seen below

What is the SINGLE most likely diagnosis?


A. Gout
B Acute limb ischaemia
C Deep vein thrombosis
D Cellulitis
E Peripheral artery disease
it is against copyright taw to take photos or distribute itfis content . May 2023
12
it is against copyright taw to late photos or distribute this content May 2023

18 A 34 year old woman presents to the Accident & Emergency department


with increasing shortness of breath over the past two days. She is also 28
weeks pregnant On examination she appears short of breath and unable
:

to complete full sentences. There is generalised wheeze throughout her


chest Her respiratory rate is 30 breaths per minute and oxygen saturations
94%. High flow oxygen with nebulised salbutamol is initiated . Treatment is
escalated to include intravenous hydrocortisone and nebulised ipratropium,
but she continues to suffer with shortness of breath and wheeze. What is
the SINGLE most appropriate next step in management ?
A Caesarean section
B CTG
C Intravenous magnesium sulphate
D Intubation
E Intramuscular betamethasone for fetal lung maturation

19 A 45 year old woman presents with sudden onset of pain and swelling in
her left calf and foot. Her main concern is that she has had a previous deep
vein thrombosis 4 years ago which requires 6 months of anticoagulation.
She denies any shortness of breath or fever On examination there is
noticeable swelling of the left calf and foot , with pitting oedema. There is no
increased warmth of the left leg Her temperature is 37 TC and a heart rate
of 80 beats/minute What is the SINGLE most appropriate investigation?
A D-dimer
B Ultrasound doppler
C. Venogram
D Joint aspiration
E Uric acid

It is against copyright faw to tote photos or distribute this content May 2023
13
tt is against copyright taw fo late phofos or distribute this content . May 2023

20 . A 37 year old woman presents with recurrent sinusitis and nosebleeds


requiring repeat courses of antibiotics. Nasal endoscopic assessment
reveals the presence of nasal mucosal crusting and ulceration. She also
complains of a cough where she has recently brought up blood. Urine dip
conffrms the presence of protein Blood tests reveal a positive c- ANCA
staining pattern What is the SINGLE most likely diagnosis?
A . Granulomatosis with polyangiitis
B Eosinophilic Granulomatosis wrth Polyangiitis
C. Sarcoidosis
D Goodpastures syndrome
E Systemic lupus erythematosus

21 A 55 year old woman presents to the Emergency Department with


sudden onset chest pain following a car accident She sustained trauma to
the chest duhng the accident hter neck veins are distended and her heart
sounds are faint. Her extremities feel cold and clammy. She has a
temperature of 38.2 *0, heart rate of 110 beats/mmute, blood pressure of
85 /55 mmHg, respiratory rate of 30 breaths/minute and an oxygen
saturation of 94%. Intravenous fluids have been started. What is the
SINGLE most appropriate management?
A Pericardiocentesis
B Antibiotics
C Non-steroidal anti-inflammatory drugs (NSAIDs)
D Thoracocentesis
E Fluid restriction

it is against copyright taw to take photos or distribute this content . May 2023
14
It is against copyright law to lake photos or distnhute this confetti May 2023

22 A 71 year old woman is referred to the two week wait gynaecology dinic
following an episode of vaginal bleeding on wiping. She denies any pain,
abnormal vaginal discharge or recent weight loss. Her last cervical smear
was 6 years ago , which was normal. On examination the skin of the vulva
;

and vagina appears thin and dry but with no focal abnormalities The cervix
is normal on speculum A transvaginal ultrasound demonstrates an
endometrial thickness of 2mm What is the SINGLE most appropriate
management ?
A Topical oestrogen
B Hysteroscopy and endometrial biopsy
C. Vulval biopsy
D. Cervical smear
E Topical oestrogen and progesterone

23
*
A 33 year old man attends the GF surgery with a cough for the past 6
weeks. He has shortness of breath and wheezing. He denies haemoptysis.
He is a heavy smoker but otherwise has no significant medical history He
has a temperature of 38.2 cCn heart rate of 90 beats/minrite, blood pressure
of 130/80 mmHg , respiratory rate of 16 breaths /minute and an oxygen
saturation of 93%. What is the SINGLE most appropriate action?
A Request an outpatient chest x-ray
B Request an outpatient CT pulmonary angiogram
C Admit to the hospital
Administer oxygen
E Perform a blood culture

It is against copynght law to lake photos or distribute /hlis content . May 2023
15
It is against copyright law to lafte photos or distribute ifti? content May 2023

24 A 12 year old boy is brought to the hospital with prolonged bleeding


following a tooth extraction. He has no past medical history. A clotting
screen demonstrates a prolonged bleeding time and activated partial
thromboplastin time, with reduced Factor VIII levels. Prothrombin time and
platelet count are normal. You note that his grandfather takes warfarin for
atrial fibrillation What is the SINGLE most likely diagnosis?
A von Wiiiebrand disease
B Haemophilia A
C. Haemophilia B
D Henoch-schdnlein purpura
E Warfarin overdose

25 . A 55 year old woman has been having pajn on the outer side of her right
hip . She describes it as aching and has been worsening over the past few
weeks. The pain is specifically at the bony prominence on the lateral aspect
of the hip. The pain is worse When she lies on her side or when she goes
for a walk. Occasionally, the pain radiates to the lateral aspect of the thigh.
She used to walk daily for at least an hour but has stopped this due to the
pain. She denies any trauma to the hips and has not had a recent fever On
examination , she is tender on palpation over the gluteus medius tendon.
There is pain on resisted active abduction and resisted internal rotation of
the affected hip joint . What is the SINGLE most likely diagnosis ?
A Osteoporosis
B Bursitis
C Osteoarthritis
D Sciatica
E Neck of femur fracture

It is against copyright law to lake photos or distribute this content May 2023
16
it is against copyright iaw to take photos or dfisPibufe itus content May 2023

26 A 26 year old man has been feeling unwell for the past two days with a
flu-like illness, including fever fatigue and generalised malaise. He is
brought into the Accident & Emergency department by his housemate as he
has now become confused and has a purpuric rash over his tower limbs. A
urine dipstick confirms the presence of protein and blood. Blood tests
reveal:

White blood count 17.7 x 10VL (4-11)


Haemoglobin 155 g/L (115-160)
Platelets
Creatinine
47 x It /L (150-400)
143 mmol/L (70-150)
^
Lactate dehydrogenase (LDH) 340 U/L ( 70-250)
Blood smear shows schistocytes
or > L

What is the SINGLE most likely diagnosis ?


A Haemolytic uraemic syndrome
B Thrombotic thrombocytopenic purpura
C Henoch-SchonEein purpura
D Autoimmune haemolysis
E Lupus nephntis

it is against copyright isw to fate photos or distribute this content May 2023
17
tt is against copyright taw to tofce photon or tfirfr& t/te ffc 3 content May 2023
*

27 . A 21 year old female university student attends the GP with a 8 day


history of a sore throat, fevers, myalgia sweats and reduced appetite. On
:

examination , her temperature is 39°C and heart rate 87 beats per minute
Her throat examination reveals whitewash exudates on her tonsils. She has
bilateral posterior cervical lymphadenopathy. What investigation will help
confirm the diagnosis ?
A Cytomegalovirus serology
B Throat swab
C Paul Bunnel test
D Sputum culture
E Liver function test

28 A 29 year old woman who underwent an emergency caesarean section


10 days ago presents with fever, righlbraast pain and tenderness. Her right
nipple is found to be cracked andsfcaly, and there is a hard 3 x 3 cm lump
on the right breast. What is;tfie SINGLE most appropriate management ?
A Oral clindamycin
B Warm compress
C Brest pump
D Stop breastfeeding
E Oral fluctoxacillin

tt is against copyright taw to take photos or dfstotute tfifs content May 2023
18
It is against copyright iaw to late photos orttislnbutB this contem May 2023

29 A 70 year old man presents to the Emergency Department with acute


chest pain with associated shortness of breath on exertion. He has also
been excessively tired lately and has had some rapid weight loss over the
last 6 months. He has a temperature of 37.7°C , heart rate of 107
beats/minuten blood pressure of 110/68 mmHg, respiratory rate of 25
breaths/ minute and an oxygen saturation of 89% What is the SINGLE most
appropriate investigation to confirm the diagnosis?
A EGG
B Chest x-ray
C CT abdomen and pelvis
D CT pulmonary angiogram {CTRA )
E MRI scan of the chest

vc
30 . You are called to see a 65 year old woman with worsening shortness of
breath. She is known to have a chronic condition requiring home oxygen
therapy An arterial blood gas shows:
pH : 7 36 [7.35 - 7.45]
p 02; 9.1 kRa [10 - 14]
pC02 : 7.9 kPa [4.5 - 6 0]
HCO 3-; 29 mmol/L [22 - 26]
What does the arterial blood gas demonstrate?
A . Mixed Acid-Base disturbances
B Type 2 respiratory failure, metabolic acidosis
C Fully compensated respiratory acidosis
D Partial compensated respiratory acidosis
E Metabolic alkalosis

it is against copyright iow to tote photos or distribute Ihw corftsrt May 2023
19
it rs ags /nsf copyright iaw to take photos or distribute tftiis confess . May 2023

31 A 71 year old man has memory problems with a deterioration of mobility


over 2 years. It was insidious in onset wrth gait disturbances being the first
feature. He has urinary incontinence a few times a week. On examination ,
he has an unsteady gait with shuffling An MRI scan of his head was
organised What is the most likely feature expected to be seen on the MRI
scan?
A . Cortical tumour
B Cortical atrophy
G Small vessel ischaemic changes
D Enlarged ventricles
E Increase in size of substantia nigra

32 A 81 year old man is brought into the Acctdent & Emergency department
with crampy abdominal pain and looserstools with blood Ichaemic colitis is
suspected Which area of the bowel rs most likely affected^
A. Terminal ileum
B. Caecum
C Rectum
D Splenic flexure of the colon
E Hepatic flexure of the colon

tl rs against copyright ta w to iaAe photo a or dfsfrt ute this content . May 2023
20
tt rs aga /nsrcopyngJrf tow to late photos ordt&ribute this content. May 2023

33 A double blinded study recruited 4000 patients with heart failure and
reduced ejection fraction of <40%. One group was randomised to take a
new drug and the other took a placebo. The primary outcome was
cardiovascular death or hospitalisation for worsening heart failure. Over a
period of 12 months, the number needed to treat was 20. What is the
meaning of the number 20?
A 20 people would need to take the new medication for one person to
benefit
B 20 people out of 4000 people benefited from this study
C 20 people did not have heart failure
D 20 people had a bad outcome from this study
E Patients needs to take the new medication for 20 days to see a benefit
o- -
cP
34 . A 23 year old man presents to A&E with a persistent nosebleed He
denies any trauma to the nose and has been applying pressure with little
success in stopping the bleeding He has previously attended on two
SL
occasions with a similar complaint. He has no other significant past medical
history, but states that his mother died of a bleed on the brain in her 50 s.
On examination, there are purple lesions on his lips and finger tips which
blanch with pressure A full blood count and clotting studies are reported as
normal What is the SINGLE most likely diagnosis?
A Infective endocarditis
B Limited systemic sclerosis
C. Hereditary hemorrhagic telangiectasia
D . Haemophilia A
E Von Willebrand disease

ttis against copynght iaw to late photos or distribute this content May 2023
21
it is against copyright iaw to lake photos or distribute this content May 2023

35 A 56 year old man with chronic heart failure with reduced ejection
fraction continues to have breathlessness. He is on the maximum tolerated
dose of enalapril and bisoprolol. He has an ejection fraction of 29%. His
heart rate is 70 beats/minute What is the most appropriate medication to
prescribe?
A Spironolactone
B Ramipnl
C Candesartan
D Adenosine
E. AmEodipine

36 A study is designed to assess the relationship between earEy-childhood


stress and the likelihood of later developing posMraumatic stress disorder
(PTSD) Two groups of patients were identified , those with PTSD and those
without Data was then collected oh any history of early life stress (e g . T
abuse, neglect, trauma) toy, each patient in both groups The data is then
compared in the two groups to determine if there is a relationship between
early life stress and the risk of developing PTSD. What type of study is
being described?
A Cross-sectional study
B Randomised control trial
C . Case- control study
D Prospective cohort study
E Retrospective cohort study

tt is against copyright taw to take photos or distribute this content May 2023
22
it is against copyright law to lake photos or distribute tfits contsnt . May 2023

37 A 46 year old woman presents to the Emergency Department with right


upper quadrant pain. Upon further questioning, she woke up in the night
feeling hot and found that her temperature was 3 S.3DC. Her girlfriend
reports that her skin appears more yellow than usual Her observations in
the department are as follows: temperature 38 4°C , pulse rate of 92 beats
per minute , blood pressure of 120/80 mmHg , respiratory rate of 12 breaths
per minute and oxygen saturation of 96% in air. Her blood test show :

Alkaline phosphatase (ALP) 390 U/L (30-150)


Alanine transferase (ALT) 102 U/L (5-35)

What is the SINGLE most useful initial investigation?


A Abdominal ultrasound
B Magnetic resonance cholangiopancreatography (MRCP )
C Cholescintigraphy (HIDA scan )
D CT Abdomen Pelvis
E Endoscopic retrograde cholangiography (ERCP)

it is against copyright iow to take photos or distribute thts content May 2023
23
it is against copyright few fo latte photos or distribute fhfe content . May 2023

38 A 36 year old man is admitted to hospital for treatment of a left lobar


pneumonia. He is known to be HfV positive and regularly injects heroin He
has also been known to share needles and so a hepatitis screen is,

undertaken wrth consent The results are as follows:


HBsAg ~ Positive
Anti- HBs - Negative
Anti'HBc - Positive
IgM Anti-HBc Positive*

What is the most likely interpretation of the patient's hepatitis serology ?


A Vaccinated
B Chronic infection
C Previous infection
D Acute infection
E . Invalid results
AS2* V
^
39 A 61 year old man presents with increasing shortness of breath on a ^- w^
background of a three-day history of a productive cough. He reports that his
other household members have also been unwell. He is a non-smoker He
is alert and oriented. His temperature is 37.4 *C, his pulse rate is 34 beats
per minute, his blood pressure is 105/75 mmHg, his respiratory rate is 20
breaths per minute, and his oxygen saturation is 94% in air What is the
SINGLE most appropriate management ?
A Arrange a chest x-ray
B Arrange urgent admission to the acute medical unit
C Home with oral amoxicillin
D Arrange a CT pulmonary angiogram
E Advice to attend the Emergency Department

it is against copyright few fo take photos or distribute this- content . May 2023
24
It is against copyright law to lake photos ot distribute this content . May 2023

40 A 41 year old woman is currently 36 weeks pregnant. This is her first


pregnancy which was achieved through in-vitro insemination (IVF ). Her
blood pressure at the start of pregnancy 128/74. She presents to the day
assessment unit with a worsening headache and floaters within her vision.
Her blood pressure is 158/98 and a urine dipstick demonstrates 2 + protein
Blood tests reveal the following
Haemoglobin 108 g/L (115- 160)
Platelets 170 x 10f /L ( 150-400)
Alanine transferase (ALT) 34 U/L (5-35)
Alkaline phosphatase (ALP) 182 U/L ( 30-150)
Creatinine 68 pmol/ L (70-150)
What is the SINGLE most likely diagnosis ?
A Gestational hypertension
B Severe pre-eclampsia
C HELLP syndrome
D Pre-eclampsia
E Chronic hypertension

41 A 41 year old woman presents to Accident & Emergency with a 15 hour


history of a severe headache with associated nausea and vomiting There
is no history of trauma She has photophobia and neck stiffness She
describes the headache as 10/10 on a pam scale She is afebrile . Her heart
rate is 80 beats/minute Her neurological examination is normal but she
looks drowsy. Pain relief has been given. What is the SINGLE most
appropriate action?
A Oral sumatriptan
B MRI scan of head
C. CT scan of head
D. Blood culture
E Lumbar puncture

It is against copynght law to latte photos or distribute UJH content . May 2023
25
It is against copyright (aw to take photos or distribute jJif s content
'
May 2023

42 . A 36 year old man has been assessed by the speech and language team
following a stroke ft has been agreed that he has a significant risk of
,

aspiration and therefore requires a raso-gastric tube for enteral feeding.


The naso gastric tube is inserted without difficulty. What is the SINGLE
most appropriate next step?
A Perform the JWhooshh test
B pH of gastric aspirate
C Start enteral feed
D Abdominal x-ray
E Chest x-ray

43 A 65 year old lady has a generalised headache and blurred vision. The
symptoms started suddenly over three days/ She describes pain developing
in her jaw muscles minutes after initialing chewfng Examination reveals
scalp tenderness on her left She has no significant medical history What
medication would be most sizable to administer?
A Prednisolone
B Aspirin
C Oral sumatriptan
D Bisphosphonates
E Nasal sumatnptan

it is agatnsl copyright (aw to take photos ordtstnbute Ihts content May 2023
26
ft is against copyright iaw (a iaVe photos or distribute this content
, May 2023

44 A 47 year oid woman presents to her General practitioner with


incontinence. She describes having to rush to the bathroom when she gets
the sensation to urinate, and occasionally losing control other bladder. She
denies any dysuria She has already tried reducing her caffeine and fluid ,
but with little success. On examination, there is no evidence of pelvic organ
prolapse and no objective evidence of unne loss on valsalva What is the
SINGLE most appropriate management ?
A Pelvic floor exercise
B Bladder retraining programme
C tntravaginal oestrogens
D Oxybutynin
E Vaginal ring pessary

45 . A 5 year old boy brought in by his mother fell unconscious in the waiting
room His respiratory rate is 9 breaths/minuter and his oxygen saturation is
83%. He is afebrile His aiiway is patent. What is the SINGLE most
appropriate action?
A Intravenous fluids
B Needle decompression
C Bag valve mask ventilation
D intubate
E . Intravenous glucose

tt is against copyright taw to lake photos or distribute this content May 2023
27
tt is against copyright (aw to lake photos or distribute bus content. May 2023

A 32 year old man presents w?th an erythematous bull’s-eye rash or his


1

46
thigh. It developed a few days following a camping trip with his father. He
also complains of arthralgia but is otherwise well He has an allergy to
penicillin. What is the most appropriate management?
A Ceftriaxone
B Amoxicillin
C Phenoxymethylpenicillin
D Erythromycin
E. Doxycycline

47 A 71 year old woman is seen in the memory clinic as she has become
more forgetful and disoriented over the past two years She lives with her
husband She has been seen lost out late at night alone She has forgotten
the names of her grandchildren She t es difficulty recognising faces and
>
people She has no known comortndrties and does not dnnk alcohol or use
drugs Her urine culture is negative. Her blood tests are unremarkable
What is the most appropriate test to perform?
A General Practitioner Assessment of Cognition (GPCOG)
B Mini-Mental State Examination (MMSE)
C . 10-point cognitive screener
D . 6-item Cognitive Impairment Test (6-CiT )
E Memory Impairment Screen (MIS)

it is against copyright law to lahe photos or distribute Ihw content . May 2023
2S
It is against copyright law to latfe photos or distribute tins content May 2023

48 . A 31 year old man with bipolar disorder has been on lithium for the past
4 years Over the last few months he has noticed an increase in the
frequency of urination and he always feels thirsty. He has recently lost 7kg
in weight. What Is the SINGLE most likely serum electrolyte picture that
would suggest nephrogenic diabetes insipidus?
A High serum sodium, low serum osmolality, high urine osmolarity
B Low serum sodium, low serum osmolality, high urine osmolarity
C Low serum sodium, high serum osmolality, high urine osmolanty
High serum sodium, low serum osmolality, low urine osmolarity
E High serum sodium, high serum osmolality, low urine osmolanty

rtfs against copyright taw to take photos or distribute this content. May 2023
29
it is against copyright law to take photos or distribute this carried May 2023

49 A 20 year old man is brought into the Emergency Department by his


father because of severe abdominal pain and constant vomiting over the
past 2 days He has become confused. He has a history of type 1 diabetes
mellitus He has a temperature of 36.7°Cn heart rate of 90 beats/minute ,
blood pressure of 130/80 mmHg , respiratory rate of 25 breaths/minute and
an oxygen saturation of 97% His venous blood test and unne test are
below ;

Blood ketones 4.1 mmol/L {<0.6)


Urine ketones +
Venous pH 7.21 (7.35 7.45) -
Venous blood glucose 50 mmol/L {<11)
Bicarbonate 16 mmol/ L (22 -26)
Sodium 148 mmol/L ( 135- 145)
Potassium 4 8 mmol/L ( 3 5-5)
^
What is the most appropriate initial management?
A Insulin
B Glucagon
C 0.9% sodium chloride
D 0 45% sodium chloride
E Bicarbonate solution

it is against copyright iaw to / ate photos or cteirilrute ffris content. May 2023
30
tt is against copyright law to lake photos or distribute this content May 2023

50 A 32-year-oEd woman develops an intensely itchy, blistering rash on her


arms , chest and back. She is currently 30 weeks pregnant. On examination,
there are multiple papules and pustules. On direct questioning, she states
that her niece developed chickenpox five days ago What is the SINGLE
most appropriate management ?
A Intravenous aciclovir
B Check varicella IgG
C Intravenous immunoglobubns
D Reassure
E Oral aciclovir

51 A 55 year old woman who is undergoing treatment for breast cancer


presents with increased thirst and confusion. She complains of having
excessive urine She has been in a very low mood for the past few weeks
She has recently become very drowsy What is the SINGLE most likely
reason for her presentation?
A Adrenal metastasis
B Diabetes insipidus
C Hypocalcaemia
D Hypercalcaemia
E Cerebral metastasis

It is against copyright law to take photos or distribute this content May 2023
31
It is against copyright law Co lake photos or distribute this content May 2023

52. A 72-year-oEd man presents with difficulty breathing which came on


suddenly over the past few hours. Prior to this he had been feeling well. His
past medical history includes type 2 diabetes meilitus,
hypercholesterolaemia, hypertension , and previous infective endocarditis
20 years ago. On examination , his blood pressure is 100/48 mmHg There
are crepitations throughout the lung fields, a raised jugular venous pulse
( JVPJ , and a pan-systolic murmur What is the SINGLE most likely cardiac
lesion?
A Mitral stenosis
B Tricuspid regurgitation
C Aortic stenosis
D Aortic regurgitation
E Mitral regurgitation

53 . A 22 year old man attends the genitourinary medicine clinic with a 4 -day
history of urethral discharge His sexual partner has recently been
diagnosed with a chlamydia infection He fears that he may have a
chlamydia infection and would like to know the complications associated
with the diagnosis. Which of the following is the most common complication
of untreated genital chlamydia infection?
A Balanitis
B Pelvic inflammatory disease
C Epididymo-orchitis
D Benign prostatic hyperplasia
E. Infertility

ft is against copyright law to taka photos or distribute this content May 2023
32
tt is against copyright law to fate photos ordtstobute Ihis content May 2023

54 A 13-year-ofd girl is noted to be short for her age. She has no problems
at school and is able to keep up with her classmates. Her periods have not
yet started. On examination, the girl has a webbed neck with a low hairline
and high arched palate Her chest is broad with wide- spaced nipples but
with no evidence of breast bud development. What is the SINGLE most
likely underlying cause?
A . Klinefelter syndrome
B Kallman syndrome
C Marfan syndrome
D Mullerian agenesis
E Turner’s syndrome

55 . A 22 year old patient , who is currently 6 months pregnant, presents


complaining of a rash on her face On examination , she has bilateral
well-defined areas of hyperpigmentation across the patient 's forehead,
cheeks , nose and upper tip- which spares the philtrum. What is the most
likely diagnosis?
A Linea nigra
B Pemphigoid gestationis
C Polymorphic eruption of pregnancy
D Pyoderma faciale
E Melasma

it is against copyright fsw to take photos or distribute this content May 2023
33
It is against copyright law to lake photos or distribute this content . May 2023

1. A 52 year old man attends his GP with a rash across his stomach and lower
back that started several days ago. He denies any itching, soreness or
burning He is otherwise well in himself. On examination, scaly red oval
patches are seen across the abdomen and alf over his back. A larger
circular patch is also noticed on his upper back. Below are pictures of the
lesions:

!
vF
<&

m
By .arnsE naltrafi.M
^. CSET-SA 3.D
What is the MOST likely diagnosis?
A Pityriasis versicolor
B. Pityriasis rosea
C Atopic dermatitis
D Pityriasis alba
E Discoid eczema

it is against copynght law to take photos or distribute this content May 2023
3
it rs against copyright law to tafoe pfrotos or distribute this content May 2023

58 A 34-year-ofd man presents to A +E following three episodes of fresh


haemoptysis. He reports having been feeling well until yesterday where he
feEt mitdly nauseous. His urinalysis shows 3+ blood 3+ protein Urine
microscopy shows red cell casts. His chest x-ray shows patchy
opacification in both lung fields His blood tests come back and he has a
creatinine of 221 gmol/ L and a urea of 21,1 mmolfl. Serological markers
are as follows
p -ANCA - Negative
c-ANCA - Positive
Arti-gEomeixilar basement membrane - Positive
What is the SINGLE most likely diagnosis ?
A EgA nephropathy
B Post-streptococcal glomerulonephritis ;
v
.
C. Systemic lupus erythematosus
D Eosinophilic granulomatosis with polyangiitis
E. Goodpasture's syndrome
^
59 A 24 year old woman presents after feeling a small, painless lump in her
left breast On examination a smooth and firm fump can be felt which is
,

about the size of a grape. It is mobile and round It is not tender on


palpation There are no skin changes above the lump The patient denies
any history of nipple discharge, or changes in breast size , shape or texture
and has no family or personal history of breast cancer. What is the most
likely diagnosis ?
A Breast cancer
B Sebaceous cyst
C. Lipoma
D Fibroadenoma
E . Breast abscess

it is against copyright law to tefrep /wtos ortfistobuto toss content May 2023
35
It is against copyright law to lake photos or distribute ATIS content
'
May 2023

60 A 28-year-oEd woman has recently had a positive pregnancy test. She


has previously been under the care of the recurrent miscarriage clinic due
to three previous miscarriages before 10 weeks gestation. She has no
personal history of venous thromboembolism. AtransvaginaE ultrasound
demonstrated normal pelvic anatomy. Serological investigations reveal:
Lupus anticoagulant - Negative
Beta:-giycoprotem 1 - Negative
Anticardiolipm igG litre - Positive { remained positive 12 weeks later )
What is the SINGLE most appropriate management for this patient?
A Aspirin alone
B Aspirin and therapeutic low-mo] ecuiar weight heparin
C Aspirin and prophylactic low-molecular weight heparin
D Prophylactic low-molecular weight heparin alone
E No additional treatment required

IS
^

it is against copyright taw to Sake photos or distribute /his content . May 2023
36
It is against copyright law to tefoe photos or distribute this content May 2023

61 A 33 year old woman attends the urgent care centre as she has
developed a lump on her left upper back over the past 2 weeks. It is very
tender, firm and mobile She is afebrile. A picture of the lump can be seen
below :

What is the most appropriate management?


A. Referral for 2 week wait haematology appointment
B Referral to surgical team for incision and drainage
C Amoxicillin and review in 7 days
D Referral to dermatology routinely
E . Discharge with ibuprofen

is against copyright (aw to lake photos or distribute bus content May 2023
37
it is against copyright (aw fa lake photos or disfnimte tfifs content. May 2023

62 . A 14-year -oEd girl with hereditary spherocytosis has been booked for an
elective splenectomy. Her baseline haemoglobin concentration is between
80-90 g/L , with going lethargy and transfusion requirements. On
examination , she is jaundiced and an enlarged spleen is palpable. When
should the pneumococcal vaccine be given to patients undergoing
splenectomy?
A 1 week before surgery
B Vaccine not required
C Just before surgery
D. 1 week after surgery
E 1 month before surgery

it is against copyright lew Fa take photos or distribute tins content. May 2023
3S
tt is against copyright law to fate / jfrotos ordist M/f& this content May 2023

63 A 54 year old man attends his General Practice to have a full health
check. He had a full workup of blood tests and a blood pressure
measurement. He has no past medical history. His blood tests were
unremarkable except for his lipid profile which reveals the following results:

Total cholesterol 6 1 mmol/L (< 5)


Triglycerides 2 6 mmol/L (0.55- 1 90)
HDL cholesterol 1.9 mmol/L (0.9- 1 93)
LDL cholesterol 2.3 mmol/L(< 2)

What should be the next steps in his management?


A Repeat blood test in 3 months
B Calculate the nsk of developing a cardiovascular event
C . Start a statin
D Refer to a lipid specialist
E. Refer to a cardiologr v
^ ^

it is against copyright taw to lake photos or distribute this content May 2023
39
ft is against copyright iaw to ! ake photos or dtstofcute this content May 2023

64 A 31-year-old man presents with worsening shortness of breath and


fatigue over the past two months. On examination, he appears pale and is
tachycardia He is referred to the hospitaJ where the following investigations
are completed :

Haemoglobin 58 g/L (130- 180)


_
MCV 89 fl (76-96)
Platelet count 237 x 10*/L (150-400)
Reticulocyte count 3% ( 0.5-2%)
Haptoglobin 0.1 g/L (0.5-2.0 g/L)
Bilirubin 49 pmot/L (3^17)
Alanine transferase 26 U/L ( 5-35 )
Creatinine 52 umol/L ( 70- 150)
c
Lactate dehydrogenase 780 U/L (70 - 2§G)
Direct antiglobulin test: Positive ^
^
Glucose-G -Phosphate Dehydrogenase activity : 14.1 units/gHb (8.6 - 18.6)
Peripheral blood film: Spherocytes
Cold agglutin titre: Negative

What is the SINGLE most likely diagnosis ?


A . Warm autoimmune haemolytic anaemia
B . Hereditary spherocytosis
C Aplastic anaemia
D Iron deficiency anaemia
E. Cold autoimmune haemolytic anaemia

ft is agatnst copyright iaw to lake photos or distribute content May 2023


40
it is against copyright law to take photos or distribute diis content May 2023

65 A 14 year old with asthma complains of increasing shortness of breath


on exertion and night time cough over the past few months. Her regular
medications include inhaled corticosteroids twice daily and salbutamol
inhaler used as required What is the SINGLE most appropriate next step in
the management of her asthma?
A Intravenous corticosteroids
B Leukotnene receptor antagonist
C Theophylline
D Intravenous magnesium sulphate
E Amoxicillin

66 A 11-year-old boy has an upper respiratory tract infection followed by a


low grade fever with erythematous macular rash , especially on the back of
the legs A few hours later, the macules evolve into purpuric lesions that are
slightly raised and do not blanch on a glass test. He also complains of a
.
headache and joint stiffness His blood tests are as follows:

Haemoglobin 123 g/L (115-160)


White cell count 3.3 x 10S/L (4- t 1)
Platelets 211 x KftL (150-400)

What is the SINGLE most likely diagnosis?


A Meningitis
B Sepsis
C Henoch- Schonlein purpura
D Idiopathic thrombocytopenic purpura
E Thrombotic thrombocytopenic purpura

it is against copyright law to taka photos or distribute this content May 2023
41
It is against copyright law to lake photos ot distribute this content . May 2023

67 A new type of gltflozin is undergoing clinical trials 2000 type 2 diabetic


patients entered the trial. Half of them were taking this new drug for a
period of one year and the other half was on a placebo. 20 of the patients
taking this new drug had diabetic ketoacidosis requiring admission while
only 10 patients taking the placebo had diabetic ketoacidosis requiring
admission , 40% of patterns in the treatment group reached their target
HbA 1 c but only 10% of patients in the placebo group reached their target
HbA 1 c after a year Which of the following is accurate ?
A The number needed to treat is 100
B The number needed to treat is 50
C. The number needed to harm is 100
D. The number needed to harm is 20
E The number needed to harm is 10

68 A 58-year -ofd woman presents with an itchy rash She has noticed it
develop over the last month On examination, she has multiple violaceous
papules over the vofar aspects of her wrists some of which have coalesced
to form a plaque A couple of the plaques have fine white lines threaded
across the surface What is the SINGLE most likely diagnosis?
A Dermatitis Herpetiformis
B Lichen Sclerosus
C. Lichen Planus
D Plaque Psoriasis
E Atopic Eczema

It is against copynght law to la /te photos or distribute fhlis content . May 2023
42
tt is against copyright iaw to lake photos or efisfnbute this content May 2023

69 An 91 year old man with malignant melanoma and brain metastasis has
worsening nausea and vomiting over the past few weeks He complains of
a severe headache. He feels weak and has been bed-bound for the past
few days. Which SINGLE medication would benefit him?
A Metoclopramide
B Haloperidol
C . Ondansetron
D Levomepromazine
E Dexamethasone

70 A 56-year-old man who works as a firefighter presents with worsening


lower back pain. It came on after he struggled to lift a piece of equipment at
work two days ago , and since then he has been in severe pain . He has
taken ibuprofen which has not heipeCUHe is also struggling to hold his
urine, often having episodes of incontinence when he laughs or coughs. On
examination , he has normal tone in both limbs but reduced power on knee
extension and ankle dorsiflexion on the left. He also has absent ankle
reflexes Palpation of his abdomen reveals a firm suprapubic mass. What is
the SINGLE most likely diagnosis?
A . Lumbar radiculopathy
B Spinal cord compression
C , Conus medullans syndrome
D Cauda equina syndrome
E. Anterior cord syndrome

ft is against copynghl iaw to take photos or distribute tfiis content May 2023
43
it is against copyright iaw to lake photos or distribute fhss content. May 2023

71 A 28 year old woman is currently 30 weeks pregnant. This is her second


pregnancy with the same partner Her first child has sickle cell anaemia and
requires regular admissions for sickle crises What is the risk of her unborn
child having the condition?
A 0%
B 25%
C . 50%
D 67%
E. 75%

72 A mother of a 3 year old boy attends her GP surgery for further


information on an incident that occurred to her son two weeks ago Two
weeks ago, her son had a sudden seizure while at home with her mother
which lasted about five minutes Ther was no tongue btting or incontinence
^
and was only mildly disorientated posLictalty Pnor to his seizure, he had
been having intermittent fevers for the past two days , due to an ongoing ear
infection. He had an uncomplicated normal, vaginal delivery and she had no
concerns about his development What is the SINGLE most appropriate
statement?
A There is no chance of developing epilepsy
B There is a 10% chance of developing a second episode
C . There is a 30% chance of recurrence
D It commonly presents above 6 years of age
E Intellect is affected

it is against copyright iaw to lake photos or distribute lhis content . May 2023
44
it is against copyright law to lake pfrotos or distnbute this content May 2023

73 A 77 year old man presents to the GP surgery with vague abdominal


pains He denies having diantioea. He has lost 10 kg over the past 12
months His abdominal examination is unremarkable . He has a 20
pack-year smoking history. His blood results show the following :

Haemoglobin 137 g/ L (130 -180)


White cell count 9 x 10*/L (4 - 11)
Neutrophils 2.1 x 10°/L (2- 7.5)
Platelets 157 x 10S/L (150-400)
Alkaline phosphatase 110 U/ L {3CM 5G)
Alanine transferase 32 U/L ( 5-35)
Gamma -glutamyl transferase 55 U/L (8 -60)
Bilirubin 15 pmol/L (3- 17)
Tissue transglutaminase IgA negative

What is the SINGLE most appropriate investigation to request?


A CT scan of abdomen
B Ultrasound abdomen
C Faecal calprotectin
D HbA1c
E Serum lipase

it rs against copyright taw to take photos or distribute this content May 2023
45
/r is against copyright law to fate photos or distribute this content . May 2023

74 A 32 year old woman discovers a small lump in her left breast. She
denies any pain, skin changes or nippfe discharge She is concerned as her
mother had breast cancer at the age of 51. On examination , there is a
smooth , rubbery mobile left breast mass It is not tender. There are no
other abnormalities of the left breast and the right breast is normal
Histology following fine needle aspiration demonstrated fibrous tissue and
glands What is the SINGLE most likely diagnosis
A Fibroadenoma
B Breast cancer
C Fat necrosis
D Fibrocystic breast changes
E Phyllodes tumour

75. A 65 year old man presents with bone pain and stiffness jn hts hips and
lower back for the past few months the pain is present even at rest Blood
tests reveal the following: .

Serum corrected calcium 2.5 mmol/L (2.1-2.6)


Phosphate 1.1 mmol/L (0.8-1.4)
Serum alkaline phosphatase (ALP) 1195 U/L ( 30-150)
Serum alanine transferase (ALT ) 32 U/L ( 5- 35)
Gamma glutamyl transferase (yGT ) 50 U/L (8 -60)
Bilirubin 11 pmol/L ( 3- 17)

What diagnosis should be suspected first?


A Liver cintiosis
B Paget ' s disease
C Osteoarthritis
D Osteomalacia
E Myeloma

it is against copynght law to take photos or distribute this content May 2023
46
it is against copyright law fa late photos or distribute this content May 2023

76 A 14 year old girl comes requesting emergency contraception Her


current sexual partner is a youth worker at the local recreation centre. She
does not want her parents to know she is sexually active. What is the
SINGLE most appropriate action?
A Inform her parents
B Advise her about safe sex and prescribe emergency contraception
C Contact the local safeguarding officer
D Inform her that it is illegal to be having sex at her age
E Contact the police

77 A 41 year old man has a knee injury after running with his child in the
park 7 days ago He reports feeling something pull and feeling a sharp
painful snap when twisting the knee. The joint had initial swelling but it
resolved He denies any knee lockin eFf giving way On examination , there
^
is mild tender swelling on his affected knee with joint effusion. He has a
positive valgus stress test He also has an antalgic gait What is the
SINGLE most likely diagnosis?
A. Fracture of the tibia
B Patella dislocation
C Anterior cruciate ligament
D Lateral collateral ligament injury
E . Medial collateral ligament injury

it is against copynght law fa take photos or distribute this content May 2023
47
.
ft is against copyright taw to 'arte photos or distribute this content May 2023

78 A 46 year old man is involved in an accident at work involving a forklift.


The machine struck him in the chest which left him short of breath and in
significant pain. He is taken to the hospital where his breathing becomes
more laboured . On examination, he is unable to complete full sentences.
His respiratory rate is 28 breaths per minute and heart rate is 112 beats per
minute There is reduced air entry on the left side and hyper -resonance to
percussion on the nght side of his chest Hts trachea is deviated to the right .
What is the SINGLE most appropriate management?
A Urgent chest x-ray
B Needle decompression in the left 5th intercostal space mid-axillary line
C Needle decompression in the nght 5th intercostal space mid-axillary
line
D Insert a chest drain on the right side
E Insert a chest drain on the left side'

79 A 55 year old woman presents with a red, itchy pen-areofar rash on her
left breast The lesion looks scaly and is located in the nipple spreading to
the areola There is no discharge. It has been present for the past 5
months Mammography done 2 years ago was normal. What is the SINGLE
most likely diagnosis?
A Bowen's disease
B Breast carcinoma
C Melanoma of the nipple
D Fibroadenoma
E. Paget’s disease of the breast

ft rs a£a /nstcopyn(jb[ taw to /arte photos ordfstnbute this content May 2023
4E
ft is against copyright law fo take photos or distribute this content May 2023

78 A 46 year old man is involved in an accident at work involving a forklift.


The machine struck him in the chest which left him short of breath and in
significant pain. He is taken to the hospital where his breathing becomes
more laboured On examination he is unable to complete full sentences.
,

His respiratory rate is 28 breaths per minute and heart rate is 112 beats per
minute There is reduced air entry on the left side and hyper -resonance to
percussion on the right side of his chest His trachea is deviated to the right .
What is the SINGLE most appropriate management?
A . Urgent chest x-ray
B . Needfe decompression in the left 5th intercostal space mid-axillary line
C Needle decompression in the right 5th intercostal space mid-axillary
line
0 Insert a chest drain on the right side Q<p
E . Insert a chest drain on the left si e
^
79 A 55 year old woman presents with a red, itchy peri -areolar rash on her
left breast The lesion looks scaly and is located in the nipple spreading to
the areola. There is no discharge. It has been present for the past 5
months Mammography done 2 years ago was normal. What is the SINGLE
most likely diagnosis?
A Bowen's disease
B Breast carcinoma
C Melanoma of the nipple
D. Fibroadenoma
E Paget' s disease of the breast

it is against copyright law to take photos or distribute this content May 2023
48
it is against copyright (aw to lake photos or distribute tfifs content. May 2023

80 A 73 year old man undergoes primary coronary intervention for a


ST- evelation myocardial infarction. Three days post -procedure an
echocardiogram demonstrates a reduced left ejection fraction What
medication would be specifically recommended in this patient?
A Spironolactone
B Propranolol
C. Ticagrelor
D Irbesartan
E. ivabradine

31 A 65 year old man undergoes a hip replacement following a fractured


neck of the femur. Three days later he becomes agitated and confused His
hands are also visibly trembling He sees rabbits coming up from his bed
and hears them sing to tom His family:state that he drinks alcohol very
regularty What is the SINGLE mgsl appropriate management ?
A Chlordiazepoxide
B. Acamprosate
C Haloperidol
D Intravenous thiamine
E Lorazepam

82 . A 76 year old woman has been treated on your ward for a community
acquired pneumonia On the day of discharge , she hands you a large
hamper of chocolate, champagne and cheeses to distribute between the
staff. What is the SINGLE most appropriate action?
A Initially refuse , but accept if she takes offence
B Accept it with gratitude
C. Ask him to make a donation to charity
D. Ask him to donate money to the ward instead
E Politely decline the gift

it is against copyright law no lake photos or distribute this content. May 2023
49
ft is against copyright law (o lake photos or distribute this content May 2023

83 A 65 year old woman presents to the Emergency Department with her


son with acute onset confusion, abdominal pain, nausea and vomiting. She
has a past medical history of ischaemic heart disease, depression f

hypertension, and type II diabetes meflitus. Her observations are


unremarkable The following blood results are reported :

Sodium 135 mmol/L ( 135- 145)


Potassium 4 1mmol/L (3.5-5.0)
Creatinine 321pmol/L ( 70-150)
eGFR 35 mUmtn (>90)

Which one of her medications should be stopped in the first instance ?


A Enalaprit
B . Dapagliffozin
G Citafopram e@
D Amfodipine
E Atorvastatin

It is against copyright law to take photos or distribute this content May 2023
50
it is against copyright taw to take photos or distribute this camera . May 2023

84 A 44 year old woman has just had her elective herniorrhaphy and she
stayed in the hospital to undergo an alcohol detoxification programme. She
is determined to stop drinking. She used to drink 22 units of alcohol a day.
Her blood tests show the following:

Haemoglobin 129 g/ L (115- 160)


Mean cell volume (MCV) 112 fL (76- 96)
Gamma -glutamyl transferase (yGT) 180 u/ L (8-60)
Alkaline phosphatase (ALP) 170 iu/L (30-150)

What is the SINGLE most appropnate medication to reduce her cravings of


alcohol?
A Acamprosate
B. Chiordiazepoxide
C. Dtsulfiram
D Lorazepam
E. Thiamine

85 A 71 year old man is found on the floor in his apartment A wardrobe had
fallen onto him and he was lying down under the wardrobe for several
hours He has tenderness in the muscles of his lower limbs He has a fever
and feels lethargic Which blood test would likely be seen elevated?
A Sodium
B Troponin
C. eGFR
D Creatinine kinase
E pH

It is against copyright taw to take photos or di&nbute this content . May 2023
51
tf is against copyright taw to take photos or dtsmbute this content . May 2023

86 A 38 year old homosexual man presents to the sexual health clinic with
two painless penile ulcers. On examination, there is also palpable inguinal
lymphadenopathy. Primary syphilis is suspected, and the patient is given
empirical intramuscular benzathine benzylpenicillin. Two hours later the
patient develops a headache , muscle aches , fever and chills. What is the
SINGLE most likely cause of his symptoms?
A . Peniciilin allergy
B . Jarisch-Herxheimer reaction
C HIV seroconversion
D Cytokine storm
E Secondary syphilis

87 A 67 year old woman with advanced breast cancer and known vertebral
metastases takes regular oral morphin ^tor pain relief Her current regime is
^
morphine sulphate 1Omg every 4 Ifaurs for breakthrough pain relief The
palliative care team would like her to go onto a slow-release preparation.
What dose of oral morphine should be prescribed to be given twice a day?
A 10mg
B 20mg
C 25mg
D . 30mg
E . 60mg

ft is agatnst copynght iaw to Sake photos or distribute tfjfs content May 2023
52
ft is against copyright iaw to lake photos or distribute this content May 2023

88 A 74 year old man presents with double vision on looking to the left On
examination, there is failure to abduct the left eye. The pupil is normal in
size and shape. His visual acuity is 6/6 in both eyes. What is the SINGLE
most likely muscle involved?
A Superior oblique muscle
B . Lateral rectus muscle
C . Medial rectus muscle
D Superior rectus muscle
E . Inferior rectus muscle

89 A 34 year old man attends for his annual asthma review He has
developed oral thrush over the past few weeks. He uses his salbutamol
inhaler and inhaled beclomethasone as prescribed He is known to have
diabetes type 2 His last HbA 1c is 47 jmtfbl/mol (target less than 48
mmol/ mol . He has a body mass iftdex of 22 kg/nrr What is the most
appropriate next step in hisLfftanagernent?
A Reduce dose of salbutamol
B Reduce dose of beclomethasone
C Encourage use of a spacer
D Wash inhalers regularly
E Discuss diet and exercise

ft rs apa^st copyright iaw to take photos or distribute toe content May 2023
53
tf is against copyright iaw fa late p /wtos or distribute this content May 2023

90 29. A 52 year old man undergoes a coronary angiogram following a


myocardial infarction. Two days later he becomes agitated and confused .
His hands are also visibly trembling. He sees snakes coming up from his
bed and hears them talk to him . He feels the nurses are planning to use the
snakes as part of the treatment for his myocardial infarction. What is the
SINGLE most appropriate management ?
A . Chiordiazepoxide
B Carbamazepine
C Haloperidol
D Intravenous thiamine
E Lorazepam

91 A 44 year old woman presents with memory loss, poor concentration and
inability to recognize household objects ?She has right -handed involuntary
'

writhing movement which has iu$t recently started She sometimes


describes jerky, random and uncontrollable movements of her hands. She
reports occasional difficulty in walking. There is a strong family history with
similar symptoms where her father and grandmother were also affected
What is the SINGLE most likely inheritance pattern?
A No genetic link
B Autosomal dominant
C . Autosomal recessive
D X -linked recessive
E. X -linked dominant

it is against copyright law to take photos or tfistoftyto this content May 2023
54
it is against copyright law to take photos ordisaribute this content May 2023

92 A 25 year old nurse is sent home from work after developing diarrhoea
and vomiting. What advice would you give her about when she can return to
work?
A 24 hours after symptoms have resolved
B 48 hours after the symptoms have resolved
C . 48 hours following the start of her symptoms
D . After a negative stool cutlure
E Once symptoms have resolved

93. A 23 year old woman presents to her GP with a 2 months history of


constipation. She opens her bowels once every 3 days She has also
gained 12 kg in the past 3 months and finds her daily task more difficult to
do with her new onset of lethargy She describes very heavy menstrual
bleeding Her abdomen is soft and notifender on palpation . What is the
r
SINGLE most likely diagnosis?
^
"

A Grave s disease
B . Fibroids
G. Addison’s disease
D. Cushing s syndrome
E Hypothyroidism

it is against copyright (aw to take photos or distribute Zfrfs content May 2023
55
ft is against copyright few to lake photos ordistribute /his content May 2023

94 A 61 year otd electrical engineer presents with worsening lethargy He


also has noted a swelling around his right coliar bone. He usually enjoys
going to the pub with his wife every Frday, but has had to stop this routine
due to episodes of discomfort in his neck following a few glasses of wine
On examination, there is a palpable iymph node above his right collar bone
On biopsy, the cells are described as two iwrorimage nuclei with an
eosinophilic nucleolus What is the SINGLE most likely diagnosis?
A Hogkin’s lymphoma
B Chronic lymphocytic leukaemia
C Acute lymphoblastic leukaemia
D Non-Hodgkin's lymphoma
E Thyroid cancer

95 . A 26 year old man who was alcohol jhtoxicated fell and hit his head on
the pavement 2 hours ago He presents to the Emergency Department with
head pain and a bruise on frs right temple. His friends who were present
^
with him give a history of loss of consciousness right after the fall, which
lasted 2 minutes His eyes open spontaneously. He is oriented to time ,
place and person. He obeys commands. What is the SfNGLE most
appropriate imaging required?
A PET- CT scan of head
B CT scan of head
C MRI scan of head
D Skull x-ray
E No imaging required

it is against copyright few to take photos or distribute tois conterrt May 2023
55
tt is against copyright law to take photos ot tiisnibuie tfits content. May 2023

96 A 24 year old man presents with diarrhoea and abdominal pain He has a
history of Crohn s disease and this is his second exacerbation within a year.
Intravenous steroids are initiated and the decision is made to add
azathioprine to induce remission What test should be prior to starting this
medication?
A Thiopurine methyl transferase test
B Interferon-gamma test
C Mantoux test
D Methionine adenosyltransferase test
E Chest x- ray

97 A 55 year old woman with metastatic ovarian carcinoma diagnosed 3


years ago is admitted to the hospital following severe abdominal colicky
A C-V
pain for the past week She is constantly bloated and has been vomiting.
She has not passed stools in th&fJest 6 days. She normally empties her
bowels every 2 days She is usually mobile but has been in the bed for the
past few days because of nausea and pain. She takes regular morphine for
the pain. On examination , her abdomen is distended with high-pitched
bowel sounds What is the next most appropnate management?
A Nasogastnc tube
B Oesophageal stenting
C. Palliative stoma
D Percutaneous endoscopic gastrostomy (PEG)
E. Barium swallow

/f is copynght lew to lake photos or fh&nbute this content. May 2023


57
it is against copyright taw to take photos ortfjstobuto rrtis content May 2023

98 A 67 year old woman presents to A&E with severe epigastric pain


radiating to her back She also complains of shortness of breath and
heartburn Her past medical history includes chronic obstructive putmonary
disease She is tachycardia on examination Her chest x-ray is below:

What is the SINGLE most likely cause of her pain?


A . Pneumonia
B . Peptic ulcer perforation
C Hiatus hernia
D. Pleural effusion
E . Pneumothorax

it is agatfTsf copyright iaw to take photos or d / stnhute this content May 2023
53
it is against copyright taw to take photos or distribute this content May 2023

99 A 55 year old woman attends the GP surgery for her yearly diabetic
review and heart failure review. She has type 2 diabetes mellitus and heart
failure with reduced ejection fraction. She takes regular metformin , ramipril
and bisoprolol. She has been compliant with her medications. Her BMI is 32
kg/m2. Her last HbA 1 c performed 2 weeks ago was 71 mmol/mol (target
less than 48 mmol/mol) Her eGFR is 85 ml/min . What is the most
appropriate management?
A Add pioglitazone
B Revisit advice on diet and exercise only
C Add empagliflozin
D Add spironolactone
E Add furosemide
C
100 A 67 year old woman presents to her general practitioner with pam and
stiffness in the small joints of both hands On examination, there are firm
nodules over the distal interphalangeal joints of the 1 st and 2nd fingers An
x-ray of her hands confirms the presence of osteophytic lumps at the joint
margins and subchondral cysts. What is the SINGLE most likely diagnosis?
A Rheumatoid arthritis
B Psoriatic arthritis
C Seronegative arthritis
D Gout -Induced Arthritis
E Nodal arthritis

it is against copynght law to take photos or distribute this content May 2023
59
it is against copyright taw to take photos or distribute this content May 2023

99 A 55 year old woman attends the GP surgery for her yearly diabetic
review and heart failure review. She has type 2 diabetes mellitus and heart
failure with reduced ejection fraction. She takes regular metformin , ramipril
and bisoprolol. She has been compliant with her medications. Her BMI is 32
kg/m2. Her last HbA 1 c performed 2 weeks ago was 71 mmol/mol (target
less than 48 mmol/mol) Her eGFR is 85 ml/min . What is the most
appropriate management?
A Add pioglitazone
B Revisit advice on diet and exercise only
C Add empagliflozin
D Add spironolactone
E Add furosemide
C
100 A 67 year old woman presents to her general practitioner with pam and
stiffness in the small joints of both hands On examination, there are firm
nodules over the distal interphalangeal joints of the 1 st and 2nd fingers An
x-ray of her hands confirms the presence of osteophytic lumps at the joint
margins and subchondral cysts. What is the SINGLE most likely diagnosis?
A Rheumatoid arthritis
B Psoriatic arthritis
C Seronegative arthritis
D Gout -Induced Arthritis
E Nodal arthritis

it is against copynght law to take photos or distribute this content May 2023
59
it is against copyright taw do take photos or distritrifte this content May 2023

101. A 26 year old woman is sent home from worfc after developing diarrhoea
and vomiting. She has a temperature of 38.9°C. She currently worlds in a
children's nursery as a chef. What advice would you give her about when
she can return to worfc?
A Return 72 hours after symptoms have resolved
B Return 48 hours after symptoms have resolved
C Return once the fever has resolved
D Return once the diarrhoea has resolved
E. Return after 7 days

102. A 57 year old woman presents with sharp central chest pain and
shortness of breath on minimal exertion The pain is worse on inspiration
She denies any cough or fever Her heart rate is 101 beats per minute,
respiratory rate 20 breaths per minute and oxygen saturations 96% on air.
An ECG demonstrates sinus rhythih. What is the SINGLE most appropriate
initial investigation?
A. Chest x- ray
B CTPA
C. V/Q scan
D Arterial blood gas
E Echocardiogram

it is agatnst copyright law to take photos or distribute this content May 2023
60
it is against copyright taw to take photos or distribute this content May 2023

103. A 21 year old man presents to the sexual health clinic after noticing an
ulcer on his penis. He denies that it causes any discomfort. He has recently
been on holiday to SpainF where he had unprotected sex with a number of
men. On examination, there is a single, shallow ulcer on the ventral side of
his penis. What is the most appropnate medication to prescribe?
A Flucloxacillin
B Acetic acid
C Raltegravir
D Ciprofloxacin
E Benzathine penicillin

104 A 41 years old nurse presents to her general practitioner with complaints
of double vision in her left eye It commonly.cccurs after a night shift while

^
watching tv and improves after sieeping he also commonly finds that she
struggles to brush her hair at night - due to weakness in her arms She
denies any headache, muscle tenderness or visual changes. On
examination , there is full range of movement of the extra-ocular muscles
and upper timbs. There is no focal abnormality in tone or power of her
upper limbs. On sustained upward gaze, left-sided ptosis is noted. What is
the SINGLE most appropriate investigation?
A MRI head
B Acetylcholine receptor antibodies
C Anti-Jol antibodies
D Evoked potential test
E CT head

it is against copyright law to take photos or distribute this content May 2023
61
it is against copyright law to fsfre pftofns or dte&rtfute tons content May 2023

103. A 21 year old man presents to the sexual health clinic after noticing an
ulcer on his penis. He denies that it causes any discomfort. He has recently
been on holiday to Spain, where he had unprotected sex with a number of
men. On examination, there is a single, shallow ulcer on the ventral side of
his penis. What is the most appropriate medication to prescribe?
A Flucloxacillin
B Acetic acid
C Raltegravir
D Ciprofloxacin
E. Benzathine penicillin

104. A 41 years old nurse presents to her general practitioner with complaints
of double vision in her left eye It commonly.occurs after a night shift while
watching tv and improves after sieepin®?She also commonly finds that she
struggles to brush her hair at night due to weakness in her arms She
denies any headache, muscle tenderness or visual changes. On
examination , there is full range of movement of the extra-ocular muscles
and upper Eimbs. There is no focal abnormality in tone or power of her
upper limbs On sustained upward gaze, left-sided ptosis is noted. What is
the SINGLE most appropriate investigation?
A MRI head
B Acetylcholine receptor antibodies
C Anti-Jo1 antibodies
D Evoked potential test
E CT head

it is agafnst copyright law to fate pJwftjs or distribute this content . May 2023
61
It is against copyright law fo fate photos or distribute this content May 2023

105. A medical student performs a neurological examination on a patient. He


attempts to elicit the ankle jerk reflex Which of the following nerve roots is
associated with the ankle jerk reaction?
A L1
B L2
C. L 3
D L4
E S1

106. A 43 year old woman attends the day surgical unit fora diagnostic
laparoscopy for chronic pelvic pain. The nurse completes her
pre-assessment checks , including observations and a urine test. The
urinary pregnancy result returns as positive, The patient denies the
possibility of her being pregnant and states she has not had recent sexual
intercourse What is the SINGLjErriost appropnate next step?
A Repeat urine pregnancy test
B Check serum beta human chorionic gonadotropin
C. Proceed with the laparoscopy
D . Perform a pelvic ultrasound
E. Cancel the operation

107 A 2 year old child is brought into the Emergency Department with severe
dehydration The paediatric doctors have failed to insert a peripheral
intravenous line. What would be the next most appropriate method to
deliver fluids for fluid resuscitation?
A Subcutaneous
B Intramuscular
C. Oral
D. Intraosseous
E Intraarterial

it is against copyright taw to take photos or distribute this- content . May 2023
62
it is against copyright iaw to take photos or distribute this content May 2023

56 A 33-year-old obese woman has been referred to Endocrinology with a


clinical suspicion of Cushing s syndrome. She complains of generalised
weakness , weight gain and easy bruising Her blood pressure is 14S/92.
She has a background of poorly controlled asthma for which she has
required multiple admissions to hospital Her current medication include
inhaled salbutamol , ipratropium, beclomethasone and montelukast with
2Gmg oral prednisolone daily What is the most common cause of Cushing's
syndrome?
A Ectopic ACTH
B Exogenous corticosteroids
C Adrenal adenoma
D Pituitary adenoma
E Pseudo Cushing's syndrome o’ ^
' •o '
57 A 5 year old boy with a suspected diagnosis of asthma is brought into the
Emergency Department by his parents with worsening shortness of breath
They have been giving him salbutamol via a spacer at home with no
improvement. On initial assessment, he is too breathless to speak His
observations are as follows: temperature is 36.5 °C, pulse rate 144 beats
per minute, blood pressure 95/67 mmHg. respiratory rate 41 breaths per
minute and oxygen saturation 93% in air What is the SINGLE most
appropriate initial intervention?
A Administer intravenous epinephnne
B fntubate
C Administer high-flow oxygen via a non-rebreather facemask
D. IV magnesium sulphate
E IV aminophylline

It is against copyright taw to lake photos or dFstofrtrte this content May 2023
34
tt is against copyright (aw to lake photos or distribute bus content. May 2023

110 A 30 year old man is brought into the emergency department following a
road traffic accident. He is unable to move his right arm and is experiencing
significant pain over the shoutder. On examination. his ami is held in an
abducted and externally rotated position, with loss of the normal contour of
the deltoid He is unable to lift his arm and there is lack of sensation over
the deltoid muscle What is the SINGLE most likely nerve affected?
A . Radial nerve
B Ulcer nerve
C Musculocutaneous nerve
D Axillary nerve
E Lower subscapular nerve

111 , A 41 year old man was found in the hospttat hallway unconscious. The
anaesthetist quickly checks hrs airwayiHls airways are patent and he is
breathing normally His blood pressure is 120/80 mmHg and heart rate is 80
beats /minute What is the SINGLE most appropriate management?
A Check full blood count
B Perform a CT scan of the head
C Check capillary glucose
D Check venous glucose
E Perform an arterial blood gas

ITJS agatfjsr copyright law to lake photos or dtstofruto Ihts content. May 2023
64
tt is against copyright (aw to lake photos or distribute bus content. May 2023

110 A 30 year old man is brought into the emergency department following a
road traffic accident. He is unable to move his right arm and is experiencing
significant pain over the shoutder. On examination. his ami is held in an
abducted and externally rotated position, with loss of the normal contour of
the deltoid He is unable to lift his arm and there is lack of sensation over
the deltoid muscle What is the SINGLE most likely nerve affected?
A . Radial nerve
B Ulcer nerve
C Musculocutaneous nerve
D Axillary nerve
E Lower subscapular nerve

111 , A 41 year old man was found in the hospttat hallway unconscious. The
anaesthetist quickly checks hrs airwayiHls airways are patent and he is
breathing normally His blood pressure is 120/80 mmHg and heart rate is 80
beats /minute What is the SINGLE most appropriate management?
A Check full blood count
B Perform a CT scan of the head
C Check capillary glucose
D Check venous glucose
E Perform an arterial blood gas

ITJS agatfjsr copyright law to lake photos or dtstofruto Ihts content. May 2023
64
It is against copyright taw to take photos or distribute this content May 2023

112 A 56 year old woman presents to her general practitioner with


troublesome hot flushes. Her last period was 3 years ago She finds that
she struggles to steep due to the hot flushes and is irritable throughout the
day. She is keen to explore treatment options Her past medical history
includes a deep vein thrombosis 15 years ago which required 6 months of
warfarin treatment What is the SINGLE most appropriate management ?
A Oestrogen- only hormone replacement
B Oral oestrogen and progesterone hormone replacement
C Topic oestrogen and oral progesterone hormone replacement
D Advise against treatment
E Venlafaxine

113 A 30 year old woman presents with a sudden onset , severe headache for
the past 3 hours, which developed while driving The headache is mainly
around the left side of her head d is associated with photophobia. A
similar episode occurred a week ago and again a month ago. The pain in
the previous episodes lasted 5 hours before it completely resolved She has
a normal neurological examination What is the most appropriate
management?
A Nasal sumatriptan
B Propranolol
C . Topiramate
D CT scan of the head
E. Oral sumatnptan

ft is against copyright law to lake photos or distribute this content May 2023
65
it is against copyright law to lake photos ordt&nbute this content May 2023

114 A 59 year old woman presents to her GP surgery with a 6 month history
of extreme fatigue and shortness of breath on exertion. She is unable to
climb a single-story house without feeling short of breath. On examination,
she appears pale. Routine blood tests are performed , which reveal the
following:

Haemoglobin 91 gft (115- 160)


Mean cell volume (MCV) 75 fL ( 76-96 )
Platelets 650 x 1D5/L (150 400)
White cell count 60 x 10S /L (4-11)
-
Ferntin 167 ng/ml ( 20-300)

Peripheral blood film shows maturing myeloid cells What is the SINGLE
most likely diagnosis?
A . Iron deficiency anaemia
B Acute lymphoblastic leukaemia
C Essential thrombocytopenia
D Chronic myelogenous leukaemia
E Sickle cell crisis

it is against copyright taw to take photos or distribute this content May 2023
66
it is against copyright IBW to lake photos or distribute fhss content May 2023

115 A 64 year old man attends the Emergency Department feeling


lightheaded His blood pressure is 95/60 mmHg. Below is his ECG :

E iffml mimr
"
l

*
f 11' .
i

' jL
i r r
tf
'r
*r TU. tuuia
*

- mi — •

;3f f l

ii till
: ziq:piH. =
::
-"- MiTF -rFrf -i Pf tt - - 5
'
tFn
- ri r -
’ 11 TI T
1 ‘
r
^

~- mu-
An enlarged image can be seen below:
i TIT r ] r ri TS
. • •
: ;-; mi ii ; i I r ;i •i

.
i i
" 1

: ImimirnitmT

;;:

Im .
i; ; j;:; j i T j rHtHrrH.

: .r rr .u.
^
liZULiiaiZili
'I " I" "
..
"!
" " " "

"
"
m^
" "
7
" *

..;,L
" * 1
'1
""" """"
_
I Mn1 '
: : j !:: •

-
" "
: :•
* TM » "

. | L ,, LL
II HI
What is the SINGLE most appropriate medication to offer him on
discharge?
A Aspirin
B Rivaraxaban
C Fondaparinux
D Warfarin
E No medication

it is against copyright law to take photos or distribute Ihts eortfe/rf. May 2023
67
It is against copyright law fo fate photos or dfStribiJte this content May 2023

116 A 49 year old man complains of worsening stabbing pain on the sole of
his right foot over the penod of months. He describes the feeling like a
' small pebble is stuck under his foot when he walks”. On examination, there
is tenderness of the intermetatarsal space between the second and third
metatarsal heads of the right foot. What is the SINGLE most likely
diagnosis?
A Plantar fasciitis
B Osteosarcoma
C Morton’s neuroma
D. Stress fracture
E Athlete's foot

117 A 52 year old male with a 1S pack-year history presents to his general
practice with a 6 month history of progressive dyspnoea, cough and
increased sputum production He denied any paroxysmal nocturnal
dyspnea or any constitutional symptoms. On examination , there is a
bilateral wheeze. There is no evidence of peripheral oedema. He is sent for
spirometry, which reveals the following results: Post-bronchodtlator
spirometry resuEts:
FEV1/FVC ratio: 0.64
FEV1: 76%
What is the SINGLE most likely diagnosis?
A . Interstitial lung disease
B Chronic obstructive pulmonary disease (CORD)
C Pulmonary fibrosis
D Sarcoidosis
E Heart failure

it is against copyright law fo iafte photos or distribute this- content . May 2023
63
ft is against copyright iaw to take photos or distribute tfiis comers:. May 2023

113 A 34 year old Ukranian woman presents with hot flushes following
cessation of the combined oral contraceptive pill She had ovarian surgery
when she was 15 years old in Ukraine. She stopped the pif ] 3 months ago,
but has not had a period since She has had an ultrasound scan 2 weeks
ago that showed an atrophic left ovary and no visualisation of the right
ovary Blood tests reveal :
FSH 88 U/L (follicular : 3- 11, ovulatory : 5-20, luteal : 18-8 )
LH 45 U/ L (follicular: 3- 11, ovulatory: 17-110, luteal: 1-13)
Oestradiol <7 pg/mL (30-400)
Prolactin 267 mU/L (<440)
bHCG <5
TSH: 4.1 mU/L (0.5-57)
T <: 112 pmot/L (9- 18 )

What is the SINGLE most likely diagnosis?


A Post-pill amenorrhoe
B Premature ovarian failure
^^
C Polycystic ovanan syndrome
D Perimenopause
E Prolactinoma

ft is apatfpsr copynght iaw to take photos or distribute this content. May 2023
69
it rs against copyright taw to take photos or distribute this content May 2023

119 A 54 year old man attends the GP clinic with his wife. He is concerned as
he has been having consistent dry ejaculation. He can maintain an erection
He has multiple medical problems and takes several medications. Which of
the following medications is the SINGLE most likely cause of his dry
ejaculation?
A Finasteride
B Oxybutynm
C Citaiopram
D Amitriptyline
E Tamsulosin

120. A 29 year old woman has been having anxiety over the past 2 months
when going into work She has asked to work from home because of the
anxiety She avoids eating out and meeting her work colleagues due to
anxiety On average, she develops palpitations and tremors, which can last
for 30 minutes , 3 times a week, especially when she gets stressed She
would go to a quiet room and try to calm down when these episodes occur.
She is keen on trying medications to help with these episodes. What is the
SINGLE most appropriate management?
A Citaiopram
B Propranolol
C Amitriptyline
D Bisoprolol
E. Zolpidem

ff rs agatfjsf copyright taw to take photos or distribute this content May 2023
70
it is against copyright iaw to take photos or distribute iftis confer May 2023

121. A 34 year old woman who suffers from depression and anxiety has just
delivered her first child 4 weeks ago. She feels her moods are getting worse
and would [ ike to start an antidepressant. She is exclusively breastfeeding.
What is the SINGLE most appropriate advice?
A Avoid all antidepressants while breastfeeding
B Fluoxetine can be started as rt is not present in milk
C Sertraline can be started as is not known to be harmful to breastfed
infants
D The only antidepressant that is considered safe is amitriptyline
E The only antidepressant that is considered safe is citalopram

122 A 32 year old man presents to the GP surgery with increasing anxiety.
He states that he struggles to leave the house due to intrusive thoughts of
someone breakrng into his house Prior to ieavmg the house , he must check
alt doors are locked and windows are locked When he does leave the
house, he continues to doutrt Whether the doors are locked and has to
return home in the middle of the day to check them. He has several indoor
cameras in his house to watch every entrance. If he is not able to check his
phone to look at his cameras he becomes sweaty and develops tremors
,

and palpitations. What is the SINGLE most likely diagnosis?


A Generalised anxiety disorder
B Panic disorder
C Bipolar disorder
D Obsessive compulsive disorder
E Post traumatic stress disorder

it is against copyright law to take photos or distribute this contert May 2023
71
it is against copyright law to take photos or distribute bits content May 2023

123 A 17 year old college boy develops a headache, high fever, and
non-blanching pruritic rash. The medical team suspect meningococcal
meningitis When is the SINGLE most appropriate time to notify the local
Health Protection Team about this condition?
A inform after treatment
B Inform immediately on clinical suspicion
C . Inform after cultures confirm result
D Inform after lumbar puncture results are available
E. No need to inform

124. A 44 year old man has perineal pain and difficulty voiding over the past 2
months On a rectal examination, he has a tender, enlarged prostate. He
denies any rigours or fever. He is not sexuaMy active What is the SINGLE
most appropriate investigation?
A First pass urine sample
B Mid-stream urine (MSU) sample
C. Ultrasound
D PSA
E Blood cultures

125 A 61 year old grandmother attends the GP surgery with her grandson
Her grandson has informed her that she is not supposed to be driving as
she has epilepsy. She takes sodium valproate and has done so for the past
6 years What is the SINGLE most appropriate action?
A Inform the DVLA
B Ask the patient to inform the DVLA
C Refer to the neurologist
D Report to the safeguarding authority
E . Look through neurology clinic letters to find out if there has been any
recent seizures

it is against copyright law to taka photos or distribute ibis content May 2023
72
It is against copyright law to take photos or distnoute this canter May 2023
*

126. A 7 week old baby is noted to be vomiting after every feed. It is


non-bi!ious and the baby vomits immediately after feeding. On
;

examination , there is a palpable mass at the right costal margin What is the
SINGLE most likely metabolic abnormality associated with this condition?
A Metabolic alkalosis
B Metabolic acidosis
C. Hyperkataemia
D Hypematraemia
E Hypocalcaemia

127. A 55 year old man with poorly controlled diabetes mellitus type 2 f

hyperlipidaemia and hypertension has scrotal and penile pain and redness
that has developed into gangrene over the period of several days He takes
ramipril , atorvastatinf metformin and cariagliflozin He is admitted for
antibiotics and surgical debndement What is the SINGLE most appropnate
advice?
A. Stop metformin
B. Stop canagliflozin
C Stop atorvastatin
D Stop ramipril
E . Continue with current medications

it is agatnst copyright law to take photos or distribute this content May 2023
73
It is against copyright law to lake photos or distribute this content May 2023

128 A 69 year old man presents with a six month history of leg pain or
walking He describes this as a “ cramping' pain in his calves, which goes
1

away when he stops to rest and is sometimes worse at night or when


walking on an incline He has a 20-pack year smoking history. He has a
medical history of hypercholesterolemia, diabetes and hypertension. He
takes atorvastatin and amlodipine On examination, you note weak
posterior tibia! and absent dorsalis pedis pulses and an ulcer on the tip of
his right hallux with a punched-out appearance What is the SINGLE most
likely cause of his symptoms ?
A Narrowing of arteries
B Medications
C Venous embolism
D Autoimmune
E . Infection

it is against copyright taw to / site photos or distribute this content . May 2023
74
It is against copyright law to lake photos or distribute this content May 2023

132. A 10 year old boy presents to the urgent care clinic with a nosebleed
The bleeding started 1 hour ago and has not stopped. He is
haemodynamicaliy stable. What is the SINGLE most appropriate next
course of action?
A Press the base of the nose
B Press the soft parts of the nose
C ice packs and lean backwards
D Start oral tranexamic acid
E. Send to the Emergency Department

133. A 91 year old man has been admitted to the intensive care unit after a
myocardial infarction He has a background of stage 4 lung cancer and
dementia. The anaesthetic team has the opinion that cardiopulmonary
resuscitation is futile if he has a cardiac arrest . His daughter would want
him to be resuscitated What rs She SINGLE most appropriate action?
A Fill in a Do Not Resustftate' order
B Resuscitate if he has a cardiac arrest
C. Ask a legal team
D Ask an independent advocate
E Fiil out an advance directive

it is against copyright few to take photos or distribute this content May 2023
76
it rs against copyright law to tafoe pfrotos or distribute this content May 2023

134. A 33 year old pregnant woman attends the urgent care clinic with a
burning sensation when urinating over the past 3 days. She is 20 weeks
pregnant. She complains of increased urinary frequency and urgency. She
has suprapubic discomfort and left loin tenderness Her temperature is
39.1°C and heart rate is 75 beatsfrninute. She declines admission What is
the SINGLE most appropriate medication to prescribe?
A Trimethoprim
B Nitrofurantoin
C. Co -amoxiclav
D Cefalexin
E Ciprofloxacin

135 A 29 year old man with bipolar disorder has been started on lithium for
mood stability He requires serum lithium levels to ensure the drug is within
the therapeutic range He usuallyfakes his dose at 9pm. When should a
serum lithium level be
-- n

A 9 am the next day


B 12 pm the next day
C 6 pm the next day
D Before he takes the lithium dose
E Anytime after the dose

it is agamst copyright law to Mep /wtus ortfjfStotHJito toss content May 2023
11
it is against copyright taw fo Jafce photos or distribute ihts content May 2023

136. A 34 year old woman presents to the GP with painful burning around her
labia. On examination, there are multiple shallow ulcers on both labia
majora. She is sexually active but dentes having any recent intercourse as
her partner has been travelling for the past 3 months What is the SINGLE
most likely diagnosis?
A Chlamydia
B Neisseria gonorrhoea
C Herpes simplex
D Syphilis
E. Klebsiella

137 A 35 year old woman who is 28 weeks pregnant attends her general
practice complaining of constipation and anal pain Her anal pain started 3
days ago She has blood on the tissue When she wipes She has already
tried increasing her water and flbtre intake but with little relief. A rectal
examination reveals an anal fissure What is the SINGLE most appropriate
laxative to administer?
A Senna
B Ispaghula husk
C Phosphate enema
D Gfycerin suppositories
E . Bisacodyl

it is against copyright taw to take photos or distribute this content May 2023
73
it rs against copyright law to lake photos or distribute tfijs content May 2023

138. A 29 year old man is undergoing surveillance gastroscopy every 2 years


for Barrett’s oesophagus. What type of epithelial metaplasia is associated
with Barrett’s oesophagus?
A Columnar to stratified squamous
B Squamous to columnar
C. Stratified squamous to cuboidal
D Pseudostratified columnar to stratified squamous
E Squamous to transitional

139. A 23 year old woman presents with pain at the base of her right thumb.
On examination, there is thickening over the radial styloid. De Quervain’s
tenosynovitis is suspected Which of the following is the SINGLE most
appropriate test to perform?
A Dix -Hailpike manoeuvre
B Finkelstein test
C Epley manoeuvre
Assessment of Tinei s sign ^
E McMurraytest

140. A 41 year old man was involved in a road traffic accident an hour ago.
He is complaining of chest pain Shortly after he falls unconscious He has
a heart rate of 120 beats/minute, respiratory rate of 25 breaths/minute, and
a blood pressure of 70/ 40 mmHg He has faint heart sounds. What is the
most appropriate initial step?
A Crystalloid fluid intravenously
B Needle thoracocentesis
C Thoracocentesis
D Pericardiocentesis
E . Colloid fluid intravenously

tt is against copyright tow to take photos or distribute hits content May 2023
79
It is against copyright law to late photos or distribute this content. May 2023

141 A 35 year old woman complains of double vision on looking to the right.
On further questioning, the two images appear side-by- side. Her pupils are
responsive to direct and indirect light stimulus, and fundoscopy is normal
What is the SINGLE most likely nerve to be involved?
A Abducens nerve
B Trochlear nerve
C. Ocufomotor nerve
D . Optic nerve
E. Facial nerve

142. A 20 year old man was investigated for tiredness Amongst his blood
results is a femtin level of 650 pg/L (<300). tt was repeated a few weeks
later and the ievel was 969 gg/L { <300). His fult blood count and liver
function test are satisfactory What is the SINGLE most appropriate test to
perform?
A Renal function test
B Total iron binding capacity
C Transferrin saturation
D Thyroid function test
E. Hepatitis C antibodies

It is against copyright law to lake photos or distribute this content May 2023
30
a$a / jnsf copyright tew to take photos or distribute this content May 2023

143. A 49 year old woman is seen in the Emergency Department with


right-sided facia! weakness. She first noticed the weakness this morning
after waking from sleep. Her observations are unremarkable. Examination
of her cranial nerves reveals a unilateral weakness of the facial muscles in
the distribution of the VII nerve affecting the right side, including her
forehead What is the SINGLE most appropriate action?
A Arrange a CT head
B Prescribe aspirin
C Prescribe prednisolone
D Prescribe aciclovir
E Check serum glucose

144 A 31 year old primigravid woman gives birth at 40+ 5 weeks to a healthy
boy Three days later, the mom notices her newborn has become
increasingly yellow in colour and js concerned She is breastfeeding the
newborn. What is the SINGLE most appropriate action?
A Admit for observations
B Perform a bilirubin test
C Ultrasound liver
D Perform a full blood count
E Reassure

it is against copyright tew to lake photos or distifiiute ihss cwrie/rf May 2023
31
it is against copyright ia w to taAe photos or distribute this content May 2023

108. A 25 year old man from the Middle East presents with significant fatigue
and yellowing of his skin. His symptoms occurred a few days after starting
nitrofurantoin fora urinary tract infection. His haemoglobin is found to be 96
g/ L (150-180), and a blood film demonstrates Heinz bodies. Given the likely
diagnosis , what is the SINGLE most appropriate test to perform as an
outpatient ?
A G6PD enzyme activity
B Osmotic fragility test
C Haemoglobinopathy test
D Coombs test
E Oonath-Landsteiner antibody

109 A 69 year old man has sudden onset chest pain radiating to his left arm
while in a restaurant . An ambulance was rushed to the scene. An
electrocardiogram shows ST ejevsftron in leads II, Nl and aVF. Which blood
vessel is likely affected?
A Right posterior descending artery
B Left coronary artery
C. Left anterior descending artery
D Left circumflex artery
E . Right coronary artery

It is against copyright taw to Sake photos or distribute tfjis content . May 2023
63
It rs against copyright law to take photos or distribute this content May 2023

147. A 39 year old woman presents with increasing tiredness and generalised
muscle aches. She also complains of poor sleep and difficulty with her
memory, particularly brain-fog . Her symptoms appear to get worse if she
7

experiences any physical or mental stress. She describes an increased


sensitivity to pain. She has no past medical history and denies any trauma
or recent infection On palpation, the patient describes muscular pain within
the trapezius , supraspinaius and paracervical muscles bilaterally. Her
rheumatoid factor is negative and she has a normal full blood count,
erythrocyte sedimentation rate {ESR) and C -reactive protein ( CRP). What is
the SINGLE most likely diagnosis?
A Rheumatoid arthritis
B Multiple sclerosis
C Myalgic encephalopathy
D Fibromyalgia
E Polymyalgia rheumatica
^

ft is against copyright law to take photos or distribute this content May 2023
33
it is against copyright law fa Saks photos or distribute this content May 2023

148. You are developing a new blood test for breast cancer screening. You
have performed the blood test on 2000 participants aged between 16-30
who have then undergone an ultrasound and core biopsy.
What is the sensitivity of the test?

Breast cancer on No evidence of cervical


ultrasound + biopsy cancer on ultrasound +
biopsy

Positive blood 850 300


result
Negative blood 150 700
result

A 15%
B 30%
C 50%
D 70%
E 85%

it is against copyright law to lake photos or distribute this content . May 2023
34
It is against copyright law to lafte photos or distribute this content May 2023

145. A 33 year old 10 weeks pregnant woman attends the GP seeking advice
for her headaches. She has suffered from migraines for the past 10 years
and would usually take sumatriptan to terminate the attacks. She was
advised to stop taking sumatriptan by her midwife until seen by her GP.
Since stopping sumatriptan , her migraines last for 24 hours with a pain
score of 9/10 at least once a week The pain results in a lack of sleep and
an inability to work She takes paracetamol but feels they are ineffective
What is the SINGLE most appropriate action?
A. Prescribe topiramate
B Prescribe zolmitriptan
C Prescribe aspirin
D Restart sumatriptan
E Continue with paracetamol

**
146 A 76 year old man has increasing abdominal discomfort, which started a
month ago. He has had recent blood tests performed by his GP for
symptoms of fatigue. The following day, the haematology laboratory calls
the clinic to inform them that his haemoglobin is 79 g/L (130- 1SO ). He
denies any rectal bleeding. His abdominal examination is normal. What is
the SINGLE most appropriate management?
A Refer urgently to an oncology clinic
B Repeat full blood count in 6 weeks
C Request for coagulation test
D. Inform patient to attend A&E urgently for a blood transfusion
E Refer to colorectal team

It is against copyright law to take photos or distribute this content May 2023
32
tt is against copyright taw to take photos w distribute tfus content May 2023

151 A 33 year old woman has been having episodes of pain around her right
eye and a right sided headache. The pain can result in tears in her eyes.
Each episode can last a few days with a pair intensity of around 3/10 on a
pain scale She has had 3 episodes of the same pain over the past 2
months She denies any visual disturbances There is no weakness or
slurred speech with the episodes She uses paracetamol for the pain when
it occurs. What is the SINGLE most appropriate medication to use to reduce
the pain during the attacks?
A Intravenous hydrocortisone
B Oral propranolol
C NasaE sumatriptan
D Oral sumatriptan
E Topical timolol

152 A 21 year old man has stepped on.a 2cm nail in the soil near his garden
which created a deep puncture yyobnd on his foot The wound is cleaned
and closed in A&E. He has completed all his childhood immunisations.
What is the SINGLE most appropnate medication to administer?
A. Tetanus vaccination
B Tetanus booster
C Tetanus immunoglobulin
D Tetanus booster and tetanus immunoglobulin
E. No medication required

It is agatnst copynght taw to take photos or distribute this content May 2023
36
it is against copyright iaw to take photos or distribute tfiis content. May 2023

153. A 78 year old man in a nursing home is brought into A&E by one of the
carers as he has become confused over the past two days. His confusion
fluctuates during the day. He complains of pain on his head. He has a
history of dementia which was diagnosed 6 months ago. He has a
background of prostate cancer which he does not receive any more active
treatment He has a bruise on the left side of his temple His last blood test
which included HbAic, renal profile, full blood count and liver function test
was done a year ago which was unremarkable. He takes rivaroxaban for
atrial fibrillation. What is the SINGLE most appropriate action?
A Blood glucose
B Urine microscopy, culture and sensitivity
C Observe for 8 hours
D CT scan of the head
E . MRI scan of the head

154. A 77 year old man attends the Emergency Department with shortness of
breath. He is known to have heart failure with an ejection fraction of 22%.
He has basal crackles bilaterally. He has a blood pressure of 150/95 mmHg
and a heart rate of 90 beats/minute. His oxygen saturation is 83% and he
has a respiratory rate of 30 breaths/minute. A chest x -ray shows bilateral
pleural effusion What is the SINGLE most appropriate medication to
manage his symptoms'?
A . Furosemide
B . Atenolol
C Bisoprolol
D Ramipril
E Amlodipine

ft is against copynght iaw to take photos or distribute ifie content. May 2023
37
it is against copyright law to lake photos or dfsfrrtHrte this content May 2023

149. A 51 year old woman is brought into the Emergency Department with
general weakness, disorientation and vomiting She has a background
history of type 2 diabetes mellrtus. She has also been suffering from
frequent urination. On examination , the woman is severely dehydrated
Capillary blood glucose is reported as 49 mmol/L. Blood tests and a venous
blood gas reveal :

pH 7.32 (7.35-7.45)
Bicarbonate 20 mmol/ L (22-26)
Haemoglobin 161 x1G&/L (115-16G)
Creatinine 215 pmol/ L (70-150)

Urinary ketones are negative What is the SINGLE most likely diagnosis?
I
A Diabetic ketoacidosis
B Septicaemia ^
C Urinary tract infection
D Hyperosmolar hyperglycemic state
E Addison s disease

150. A 39 year old pregnant woman has a painless lump on her left breast
that developed 10 weeks ago She does not take any medications. Her
mother had breast cancer at age 45 On examination , she has a 2 cm
painless mobile lump in the upper outer quadrant of her left breast Which is
the most appropriate initial investigation?
A Ultrasound scan
B Mammography
C Magnetic resonance imaging
D CT scan
E Full blood count

ft is against copyright taw to take photos or distribute this content May 2023
S5
it is against copyright faw to take photos or distribute this content . May 2023

157. A 44 year old man was involved in a road traffic accident where he
sustained an injury to his right arm He extended his right upper extremity
against the dashboard just before the impact. He initially presented to the
Emergency Department with right wrist tenderness and lacerations of the
wrist. X-ray of the wrist showed fractures of the distal radius and scaphoid
bones. There was also a lunate bone dislocation A splint was applied . 20
hours later, he represented because of worsening pain The splint was
removed and there is noticeable swelling on the hand and forearm
Capillary refill is 2 seconds in all digits. His radial pulse is normal. What is
the SINGLE most appropriate action?
A Apply compression dressing
B Take intracompartment pressure measurements
C. Ultrasound of arm
D . Reapply splint and reassure
E Administer furosemide c -*
158. A 5 year old girl is seen in the paediatric clinic with incontinence of urine.
Her mom thinks that she must have "a small bladder capacity" because she
has to go to the toilet very frequently She often gets the urge to urinate
suddenly and bends forward , trying to postpone urination Many times she
fails to make it to the toilet and ends up emptying her bladder completely.
She wakes up 3 times a night to use the toilet A urinalysis is seen as
normal Mid-stream urine cultures are negative. Which of the following is
the most likely diagnosis?
A Overactive bladder
B Stress incontinence
C. Neuropathic bladder
D Vesicoureteric reflux
E Mixed incontinence

it is against copyright taw to take photos or distribute this coolsnt May 2023
39
it is against copyright iaw fo lake photos or distribute this content May 2023

159 A 1 year old child presents to her GP with a dry rash. She was given
emollient by the GP 12 weeks ago, which she has been using as
prescribed. Below is a picture of the rash on the legs:

Her mom says the rash has been worsening for the past 4 weeks and she
has still been using the emollients. She is seen frequently scratching which
can disturb her sleep. A similar rash is found on the whole body. What
SINGLE most appropriate action?
A Clobetasone
B . Clobetasol
C Hydrocortisone acetate 1%
D Permethrin
E. Continue with emollients only

it rs against copyright law to take photos or distribute this content May 2023
90
iff against copyright law to take photos or distribute this content May 2023

160. A clinical trial was performed looking at a new drug called Sztop. It is
thought that this new drug is able to modify risk factors and reduce the risk
of an ischaemic stroke. 200 patients who have not had ischaemic strokes
before were randomised into two separate groups. One group received the
new drug, while the other group received the placebo . The participants
were followed up for 12 months
The results are below :

Developed No Totai
ischaemic ischaemic
stroke stroke

Sztop 16 100

Placebo 20 80 100
-
Total 3&# ‘
164

What is the numbers needed to treat?


A. 5
B 20
C . 25
D 50
E 100

tt is against copyright taw to Sake photos or drstTftute ihfs content May 2023
SI
tt is against copyright taw to take photos w distribute tfus content May 2023

161. An 82 year old man has been assessed by the speech and language
team following a stroke. It has been agreed that he has a significant risk of
aspiration and therefore requires a naso-gastric tube for enteral feeding.
The NG tube is inserted without difficulty. A pH indicator strip shows a pH of
5 What is the SINGLE most appropriate next step?
A Perform chest x-ray
B Perform the WhoosiV test
C. Repeat pH
D Start enteral feed
E Abdominal x -ray

162 A 6 year old boy is brought into the clinic with a progressive limp with
pain in his upper thigh The limp started a few weeks ago without a history
of trauma . He is otherwise well On examination , there is reduced abduction
and medial rotation of the left hip; The pain is in the anterior thigh An
anterior-posterior and frog-leg x-ray demonstrates flattening of the left
femoral head. What is the SINGLE most likely diagnosis?
A. Irritable hip
B Slipped upper femoral epiphysis
C Transient synovitis of the hip
D Juvenile rheumatoid arthritis
E. Perthes disease

It is agatnst copynght taw to take photos or distribute this content May 2023
92
It is against copyright law to take photos or distribute tfiis content May 2023

163 A 69 year old man is brought in by his wife He had been unsteady on his
feet for the last couple of months with frequent falls forward. He has been
having daily incontinence of urine. He has been less mobile since he injured
his back from a fall He has become more forgetful lately She wonders if he
is depressed because of the reduction in his mobility. He has a shuffling
gait MRI of the head shows enlarged lateral and third ventricles What is
the most likely diagnosis^
A Progressive supranuclear palsy
B Parkinson s disease
C. Alzheimer 's disease
D Normal pressure hydrocephalus
E Myalgic encephalomyelitis
\ C
164. A 7 year old boy has had 3 tonsitlitisepisodes in the past 12 months . 1 of
the episodes required antibiotic His mother is very concerned as he
started to have a sore throatfast night with a cough. He does not have a
fever His immunisations are up to date He has no cervical
lymphadenopathy. There are no exudates seen on his tonsils. He is
penicillin allergic What is the SINGLE most appropriate management?
A Refer to ENT for tonsillectomy
B Prescribe phenoxymethylpenicillin
C Prescribe clarithromycin
D Swab the throat
E Reassure

it is against copyright law to taka photos or disfnfu/te content May 2023


93
It is against copyright taw to tafia photos or distribute this content May 2023

165. A 71 year old man presents to the GP surgery with a history of loss of
balance and difficulty in walking He developed a gradual decline in his
balance over the past 3 months. He has been falling over the past month.
He has minor bilateral tremors and neck rigidity. He has a restricted
downward gaze on examination His MRI scan shows midbrain atrophy with
a hummingbird sign What is the likely diagnosis?
A . AJzheimer s disease
B Shy-Drager syndrome
C. Idiopathic Parkinson s disease
D Progressive supranudear palsy
E Pick's disease

166 A 71 year old man is being investigated for chronic back pain and lower
limb pain An x -ray of his spine is reported to be normal An MRI
demonstrates multiple lesions in Jhe skeleton suggestive of metastatic
disease What is the SINGLE most likely site of the primary?
A. Skin
B Brain
C Testicle
D Anus
E . Prostate

tt is agamst copyright taw to tafia photos or distribute this content May 2023
94
It is against copyright law to lake photos or dfsfrS>ute this content May 2023

167. A 65 year old man has been having nausea and vomiting for the past 2
weeks. He is under the palliative care team as he has terminal prostate
cancer with peritoneal metastasis. He has not opened his bowels over the
past 12 days. He takes morphine for pain. He feels constantly full and has a
decreased appetite His blood test shows that his renal function and
calcium levels are within normal ranges On examinationr he has soft
sluggish bowel sounds His abdomen is nontender What is the SINGLE
most beneficial antiemetic to use?
A Haloperidol
B Metoclopramide
C Cycfizine
D Domperidone
E Prochlorperazine
of *
168 A 29 year old woman has been feeling very energetic and has been
overspending. She is unabJe to sleep throughout the night as she keeps
thinking about her life. She is euphoric, full of energy and has just moved to
Canada to start a new business. She speaks quickly and does not like to be
interrupted. What is the SINGLE most appropnate diagnosis?
A Borderline personality disorder
B Mania
C Bipolar affective disorder
D Antisocial personality disorder
E Schizophrenia

It is against copynght law to take photos or distribute content May 2023


95
it is against copyright few to lake photos or distribute this content May 2023

169. A 33 year old man has had pale patches on both his hands which he
developed 4 years ago. He has a background of hypothyroidism. The
patches are not itchy and not painful. Befow is a picture of his hand.

What is the SINGLE most likely diagnosis ?


A Psoriasis
B. Vitiligo
C Lichen sclerosus
D Pitynasis alba
E . Pityriasis versicolor

it rs agswisrcopynoftf few to take photos or distribute content May 2023


56
it is against copyright law to take photos or distribute this content May 2023

170 A 26 year old man attends the Emergency Department with abdominal
pain. He has a history of ulcerative colitis He has been having episodes of
severe diarrhoea over the past 4 days. In the last 24 hours, he had 17
episodes of diarrhoea. On examination, he is tender at the left iliac fossa .
He has a temperature of 33.1°Cr heart rate of 112 beats/minute , respiratory
rate of 20 breaths per minute and an oxygen saturation of 99%. What is the
most appropriate management?
A Oral prednisolone
B Topical mesalazine
C. intravenous antibiotics
D intravenous adrenaline
E tntravenous hydrocortisone

171 A 33 year old woman presents to the GFtsurgery with a 2 cm small


smooth , firm mass in her left breast Tfttfmass is non - tender and there is no
nipple discharge or nipple retraction She first noticed the lump 4 weeks ago
and thought it would go away initially however, it has remained. She has no
family history of breast cancer. She is a non-smoker and does not have any
medical problems. There are no associated skin changes in the breast. The
lump is slightly mobile on examination What is the SINGLE most
appropriate action?
A Refer to obstetrics and gynaecology cfinfc to be seen within 2 weeks
B Refer to breast clinic to be seen routinely
G Refer to breast clinic to be seen within 2 weeks
D Organise an ultrasound of the breast
E Reassure and review in GP clinic in 4 weeks

it is against copyright taw to lake photos or distribute iftnE content May 2023
97
it is against copyright iaw to take photos or distribute bits content May 2023

172. A 19 year old man suffers a second-degree bum to his thigh and trunk
during a campfire accident. An ambulance is phoned and he is taken to the
nearest burn centre. Which of the following formulas is used to calculate the
fluid required to be administered following a bum?
A (Body area burned (%) x Weight (kg) x 8ml) over 24 hours
B (Body area burned (%) x Weight (kg) x 4ml) over 8 hours
C . (Body area burned (%) x Body mass index x 12ml) over 8 hours
D (Body area burned (%) x Weight (kg) x 8ml) over 8 hours
E. (Body area burned (%) x Weight (kg) x 2ml) over 24 hours

173. A 3 year old boy presents to the GP surgery with his mother She wants
some advice on vitamin D supplements for her and for her child She
mentioned that they both have a healthy balanced diet They both do not
have any allergies and are not taking #ny regular medications. She was
diagnosed with coeliac disease 2 years ago What is the most appropriate
advice?
A No vitamin D supplement required
B Vitamin D 400 IU for the child only
C Vitamin D 400 IU daily for the child and 800 IU for the mother
D Vitamin D 400 IU weekly for both mother and child
E Vitamin O 400 IU daily for mother and child

it is against copyright iaw to take photos or distribute bus content May 2023
93
it is against copyright law fo lake photos or distribute this content May 2023

174. A 39 year old man presents 5 weeks after a trip to Africa with a
headache and a fever. He has a loss of appetite and joint pains He has
been having chills and rigours over the past few days with gastrointestinal
upset He has an enlarged tender liver which is palpable at 2 cm below the
,

right costal margin , with a palpable spleen. He has a temperature of 39.5 X


His blood test results are seen betow ;

Haemoglobin 100 g/ L (130-180)


White cell count 12 x HM. (4-11)
Eosinophils 0.6 x 10*/L (0.04-0.44)
Platelets 30 x 10*VL (150-400)
Serum alkaline phosphatase (ALP) 250 U/L (30-150 )
Serum alanine transferase (ALT) 112 U/L (S-S5)
Serum urea 6 mmol/L ( 2 0 - 7)
Serum creatinine 143 pmol/L (7p- f 50)
'
eGFR 90 mL/min (>90) Jt&
'

What is the SINGLE most likely diagnosis?


A Malaria
B Legionnaires' disease
C . Schistosomiasis
D Lyme disease
E Amoebiasis

it is against copyright taiv to isfae photos or distribute tfiis cwiteffl May 2023
99
It is against copyright law to tote pfnoftw or distribute this content May 2023

175. A 31 year old woman has a very swollen lump near her anal verge The
mass is itchy and painful. The pain started a day ago . She dentes any
bleeding. The lump is seer as a 4mm diameter dark blue tender mass
adjacent to the anus. What is the SINGLE most likety diagnosis?
A Pilonidal cyst
6 Thrombosed external haemorrhoid
C . Internal haemorrhoid
D Anal fissure
E Uterine prolapse

it ;s aga^sf copyright law to lake photos or distribute tfws content May 2023
100
it :$ against copyright iaw to lake photos or distribute this content May 2023

155. A 23 year old woman complains of episodic epigastric pain over the past
3 months She describes bloating. She has episodes of diarrhoea that car
last for a few days, followed by constipation . She denies any blood in the
stool. Her temperature is 37 3°C and heart rate is 70 beats/minute She
normally opens her bowels once a day What is the SINGLE most likely
reason for her symptoms?
A . Gastroenteritis
B Dysentery
C tmtable bowel syndrome
D Ulcerative colitis
E Crohn's disease

156 A 44 year old woman attends the GP surgery for an annual review She
takes tevothyroxine for hypothyroidism: Her last blood test shows the
following:

Thyroid stimulating hormone (TSH) 13 mU /L {0.5-5.7)


Free thyroxine (T4) 3 pmol/L (9-18)

What is the SINGLE most appropnate management?


A. increase her levothyroxme
B Request thyroid antibodies
C Repeat thyroid stimulating hormone in 3 months
D Ultrasound the thyroid gland
E Decrease her levothyroxine

it rs against copynght iaw to take photos or distribute this ccwfert May 2023
3S
a$a / jnsf copyright tew to take photos or distnbule this content May 2023

177 . An 82 year old man has a 3 week history of muscle pain and fatigue with
bilateral hip and shoulder pain. His atorvastatin was stopped last week by a
previous doctor assuming his presentation may be statin-induced. He
complains that the pain has got worse and is accompanied by stiffness He
denies any weight loss, fever , visual disturbances or headaches Which of
the following abnormal blood tests would support an alternative diagnosis ?
A . Full blood count
B Antibody screen
C Rheumatoid factor
D Erythrocyte sedimentation rate
E Liver function test

it is agamsl copyright tew to lake photos or disotifute tfifs content May 2023
102
a$a / jnsf copyright tew to take photos or distnbule this content May 2023

177 . An 82 year old man has a 3 week history of muscle pain and fatigue with
bilateral hip and shoulder pain. His atorvastatin was stopped last week by a
previous doctor assuming his presentation may be statin-induced. He
complains that the pain has got worse and is accompanied by stiffness He
denies any weight loss, fever , visual disturbances or headaches Which of
the following abnormal blood tests would support an alternative diagnosis ?
A . Full blood count
B Antibody screen
C Rheumatoid factor
D Erythrocyte sedimentation rate
E Liver function test

it is agamsl copyright tew to lake photos or disotifute tfifs content May 2023
102
ft is against copyright iaw to fate photos or distribute this content May 2023

178. A 41 year old woman has been having a sore left leg over the past 3
days after an insect bite on her left shin. She describes the area as very
painful, swollen and tender to touch. She is currently 27 weeks pregnant.
She has a temperature of 38.2°C and a heart rate of 91 beats/minute.
There Is increased warmth in the area affected. A picture other limb can be
seen here :

What is the SINGLE most appropriate management?


A Erythromycin
B Phenoxymethy [penicillin
C Paracetamol
D FlucloxaciHin
E Colchicine

It is against copyright iaw to take photos or distribute this content May 2023
103
it is against copyright iaw to lake photos or distribute this content May 2023

179 A 6 year old boy presents with wheezing and shortness of breath , which
has been waking him at night. The general practitioner suspects a
diagnosis of asthma and has prescribed a salbutamol inhaler which he finds
useful. He has been using the salbutamol inhaler around 6 times a week.
What is the SINGLE most appropriate next step in management?
A Inhaled long -acting beta-2 agonist
B Inhaled corticosteroid
C. Leukotnene receptor antagonist
D Oral prednisolone
E Oral theophylline

180. A 40 year old woman is requesting long term contraception with the
lowest risk of falling pregnant. She has a new partner and is sexually active
She is a non-smoker and her blood pressure is 130/80 mmHg She has a
past history of a deep venous thrombose Her mother has breast cancer.
What is the SINGLE most appropriate action?
A Advice to use barrier methods
B Advice to use oral combined hormonal contraception
G Advise to use a Mirena coil
D Advise to use a copper intrauterine device
E Advise to use progesterone-only pill

tt is against copyright taw to take photos or distnbute this content May 2023
104

You might also like